18 Oct

You might also like

Download as pdf or txt
Download as pdf or txt
You are on page 1of 82

1 patient HTN had dry cough what you will give him ? Antitussive drug?

Thyme
2 patient planning to be pregnant and she takes levothyroxine 100 what to do?
Increase the levothyroxine dose by 30-50%
3 meaning of category D ?

4 patient has acute migraine came to ER with severe migraine headache what medication will be used ?
Ibuprofen 400 mg TID This is for mild to moderate but for severe we will give him Triptans
5 microbiology, image of antibiotic disk , what is the following is sensitive antibiotic to bacteria?

6 which one of the IG is responsible of hypersensitivity?


IgE
7 women 23 years old has UTI what is best option for her ?
Trimethoprin-sulfamethoxazole 160/800 mg BID for 3 days
8 what will do with warfarin of INR 12 ? Hold warfarin until INR below upper limit of therapeutic range and
give vitamin K orally and decrease weekly maintenance dose by
5% to 20%
9 blood pressure in infants ?
Low than adult
10 what is mean of polypharmacy ?
Use multi medication by patient
11 what is warfarin level in mechanical valve ?
2.5 – 3.5
12 what is ingredient use to prepare suppository?
Cocoa butter
13 what is CI vaccine in pregnant?
Live vaccines in pregnancy
14 type of CYP interaction between omeprazole and clopidogrel ?
CYP2C19
15 type of WBC cause immune defenses ?
Neutrophils
16 type of WBC cause antibody production?
B-lymphocytes
17 what is best drug for asthma is caused by exercise?
SABA
18 which of the following drugs requires dosage reduction in hypoalbuminemia ?
Phenytoin. The total phenytoin level will appear artificially low if albumin is low.
19 what type of infections is prevented by vaccination?
Varicella
20 what is of the following is CI in pediatric?
Tetracycline
21 what is drug most uesd for MRSA when vancomycin not effect ?
Linzolide
22 which of the following not essential amino acid ?
Essential: try this vip mall
23 using the book USP797 for which product ?
Sterile

24 antidote of diazepam ?
Flumazenil
25 Pregnant women has hepatitis B when she delivers we must give her baby ?
Hepatitis B vaccine + Hepatitis B immunoglobulins
26 Patient 65 years with renal failure?
Assess the patient medications for nephrotixics and other
27 pregnant women with pyelonephritis went to hospital given ceftriaxone what is antibiotic to go with ?
cefixime
28 Patient 26 years old with athletic foot
Treatment? Topical antifungals: terbinafine, azoles (clotrimazole, miconazole), ciclopirox, or tolnaftate)
Oral antifungal (Clotrimazole, Miconazole, Terbinafine. Ketoconazole)
29 vit C used for ?
Preservative
30 case about drug the pharmacy permit.‫ ﺗﺮﺧﯿﺺ اﻟﺪواء‬it take approve in 2009 ?
2014 .‫ﺗﺮﺧﯿﺺ اﻟﺪواء ﯾﻨﺘﮭﻲ ﺑﻌﺪ ﺧﻤﺲ ﺳﻨﻮات ﻣﻦ ﺑﺪأ اﻟﻤﻮاﻓﻘﺔ‬
31 drug expire in December 2020 what is the last day of use for it ?
31 December
32 When women start folic acid before get pregnant?
1 month before pregnancy
33 women had accident , she used anxiolytic since 5 years with phobia of supermarket , and befroe few
days she used cimetidine for acid ? What is the reason of sedation action ?

34 car driver wants antihistamine ?


2nd generation Antihistamine example ( loratidine)
35 which of following is high alert solutions?
Dextrose 20%

36 severe case of alzheimer we give ?


Donepezil alone or in combination with Memantine
37 Pharmacovigilance definition ?
The science and activities relating to the detection, assessment, and prevention of adverse effects or any other drug-related problem .

38 what is your advice for patient take fentanyl patch ?


Avoid hot baths, sunbathing and use of heat sources include(tanning beds, heating pads, hot Tubs) and avoid activities like heavy exercise

39 patient take iron what is your advice for him ?


Use vitamin C
40 DM patient needs drug for hypertriglycerdemia ?
Statin
41 drug mask hypoglycemia?
Beta blockers
42 which drug used for glaucoma ?
Timolol -first line prostaglandin analogues
43 Nurse received extra vial in the bag what should we do ?
Return to pharmacy
44 Dr wants to know about interaction between garlic and drug he should search in ?
1-Hansten and Horn’s Drug Interactions Analysis and Management 2-Drug Interaction Facts 3- micromedex or lexicomp

45 MOA for carpamzpine?


Na channel blocker
46 Plant source anticancer?
Vinca alkaloids (Vincristine, Vinblastine) & Taxanes (Paclitaxel, Docetaxel)
47 Time for drawn gentamicin?
30 minutes before 4th or 3rd dose
48 Beta blocker antidote?
Glucagon or Calcium chloride or Calcium gluconate
49 Enoxaparin for pregnancy?
Category B
50 Drug C.I in sulfa allergy?
Bactrim SMX/TMP and other medications that contain sulfa
51 Drug C.I in High K?
Spironolactone, Eplerenone, ACEs, ARBs
52 Cost effectiveness?
To identify, examine, and compare relevant costs and consequences (clinical outcomes) of competing
drug regimens and interventions.
53 Calcium give with corticosteroids long use
To prevent osteoporosis

54 SE of methotrexate?
Warning: Hepatotoxicity, Myelosuppression, Stomatitis/Mucositis.
55 SE of methadone?
QT prolongation
56 AF class of rate control
Rate Control
* Digoxin
* β- blockers
* Nondihydropyridine calcium channel blockers ( not used. In pt with HF)
*Amiodaron
57 Methotrexate category in pregnancy?
Category X
58 ACEI in pregnancy?
category C in the first trimester and category D in the second and third trimesters.
59 CrCl?

60 Drug of choice in asthma?


ICS
61 Prevalence calculation
Prevalence: number of persons who have the disease at a specified time divided by the number
of persons in population at that specified time
62 Allele definition?
Different versions of the same gene are called alleles
63 SE of lamotrigine?
Rash
64 Antidote for ACH?
Atropine
Or
if the question means anticholinergics the answer will be (Physostigmine)
65 Drug used for respiratory syncytial virus? RVs?
If it’s Rotavirus just need Supportive Care
If it’s Respiratory Syncytial virus treated by Pavilizumab
66 Infusion rate?
ml/min
67 Cl calculation

68 Vaccine for wounds

69 other name for hospital pneumonia?


Nosocomial pneumonia or Ventilator associated pneumonia
70 vaccine for DM?
Influenza vaccine annually, Pneumonia (PPSV23), Hepatitis B
71 Drug has positive int effect on heart
Digoxin
72 Aspirin-ibuprofens-quinine structure

73 Warfarin monitoring? And dose?


INR Initial 5 mg once daily for most patients or 2.5 mg once daily for sensitive patients
74 Ethics?
1-Beneficence. 2-Autonomy 3-Honesty 4-Informed Consent 5-Confidentiality 6-Fidelity
75 Memorize the schedule and the drugs, imp**

76 What principle ethics?


Justice
77 Do the following in order (from top to bottom or vice versa) “Maslow's hierarchy of needs”

78 QRS represent what? P wave represent what?


Depolarization —> Contraction
Repolarization / hyperpolarization —> Relaxation of heart chambers.

P wave= Atrium depolarization


QRS complex= Ventricles depolarization
T wave= Ventricles Repolarization
79 Ca dose in pregnancy?
1200 mg/day 1000 mg/day
80 Clozapine structure?

81 Enoxaparin dose?
Treatment 1mg/kg SC BID or 1.5mg/kg SC OD PPX: 30mg SC twice daily or 40mg once daily
82 Pneumonia management?

83 Lady has UTI and resistance to penicillin?


Ciprofloxacin
84 MOA for carpamzpine? Na channel blocker

85 Plant source anticancer?


Vinca alkaloids (Vincristine, Vinblastine) & Taxanes (Paclitaxel, Docetaxel)
86 Time for drawn gentamicin? Before the 4th dose
87 BB antidote?
Glucagon or Calcium chloride or Calcium gluconate
88 patient when pregnant was take levothyroxine 100 after pregnant what is the dose ?
meaning of category D ?
89 patient has acute migraine came to ER with severe migraine headache what medication will be used ?
90 microbiology, image of antibiotic disk , what is the following is sensitive antibiotic to bacteria?
Big circle
91 women 23 years old has UTI what is best option for her ?
Trimethoprin-sulfamethoxazole 160/800 mg BID for 3 days
92 blood pressure in infants ?
Low than adult
93 - what is mean of polypharmacy ?
Use multi medication by patient
94 - what is warfarin level in mechanical valve ?
2.5 to 3.5
95 what is ingredient use to prepare suppository?
Cocoa butter
96 - what is CI vaccine in pregnant?
97 - type of CYP interaction between omeprazole and clopidogrel ?
CYP2C19

98 Beta blocker for heart failure ?


bisoprolol
99 Pseudomonas aeruginosa in gram stain ?
A - gram positive cocci
B- gram nigative cocci
C- gram positive bacilli
D- gram negative bacilli
100 Complication of parenteral nutrition
‫ﺟﺎت ﻛﯿﺲ و ﻻب ﺣﻘﺖ اﻟﻤﺮﯾﺾ ﻓﯿﮭﺎ اﻧﺰﯾﻤﺎت اﻟﻜﺒﺪ ﻣﺮﺗﻔﻌﺔ‬
‫اﻻﺧﺘﯿﺎرات ﻛﺎﻧﺖ‬
Liver cirrhosis
Liver cholestasis
Pancreatitis
Cholestasis
102 Which electrolyte disturbance lead to digoxin toxicity
Na
K
Mg
Ca
103 How many phases of drug study until it’s approved?
1 phase
2 phases
3 phases
4 phases
104 Plant with adaptogenic effect ?
105 Plant used for cough relief?
Eucalyptus oil
106 Pt. Taking anti TNF should be tested for ?
Tuberculosis
107 Plant used to treat sunburn?
Aloe Vera
108 Plant used to treat vomiting?
Ginger
Cinnamon
Clove
109 Plant used to increase physical endurance?
Ginseng
Ginger
‫واﺧﺘﯿﺎرﯾﻦ ﺛﺎﻧﯿﺔ ﻧﺴﯿﺘﮭﻢ‬
110 Child one years ago, had immunodeficiency, he took before 4 month IVIg, and now it’s time to take MMR Vaccine
A -Give vaccine
B- don’t give because he took IVIG
C- don’t give because he has immunodeficiency
111 Doxorubicin toxicity on ?
Heart

112 ‫اﻋﺮﻓﻮ اﻟﻔﺮق ﺑﯿﻦ‬


Primers , isomers , enantiomers
‫ﺑﺎﻟﺸﻜﻞ ﺿﺮوري‬
‫ﯾﻌﻨﻲ ﻛﯿﻒ اﻟﻤﺮﻛﺒﯿﻦ ﯾﻜﻮن اﯾﺰوﻣﺮ او اﯾﺒﯿﻤﺮ ﻟﺒﻌﺾ‬
‫ﺟﺎﻧﻲ ﻣﺮﻛﺒﯿﻦ و اﻟﺴﺆال اﯾﺶ اﻟﻌﻼﻗﺔ ﺑﯿﻦ اﻻﺛﻨﯿﻦ ؟ ھﻞ ھﻢ اﯾﺰوﻣﺮ وﻻ اﺑﯿﻤﺮ‬
‫ اﺳﺌﻠﺔ ﻣﻦ دا اﻟﻨﻮع‬٣ ‫ﺗﻘﺮﯾﺒﺎ‬
113 ‫ ﺳﻨﮫ وﻋﻤﺮه ﻛﺒﯿﺮ‬٢٥ ‫ﻣﺮﯾﺾ ﻣﺘﺰوج ﻟﮫ‬
‫ وﻣﻌﺎه‬rellict dysfunction ‫ راﻓﺾ ﯾﺎﺧﺬ اﻟﻌﻼج ﻋﻦ طﺮﯾﻖ‬، inje and pill
‫ ﻣﻦ ﺿﻤﻦ اﻟﺨﯿﺎرات‬:
‫اﻟﺠﻠﺴﺎت اﻟﻨﻔﺴﯿﮫ‬
‫ ﯾﺎﺧﺬ‬topical
114 Vision 2030, interested for 3 disease , which are?
115 ‫اﻟﺤﻤﻮﺿﺔ ﻓﻲ اﻟﻤﻌﺪة ﻋﻨﺪ اﻟﻤﻮاﻟﯿﺪ واﻟﻤﻜﺘﻤﻠﯿﻦ اﻟﻨﻤﻮ‬
‫ ﻣﻦ اﻟﺨﯿﺎرات‬:
- ‫ﻧﻔﺲ اﻟﺸﻲ ﻣﺘﺴﺎوﯾﻦ‬
- ‫اﻟﻤﻮاﻟﯿﺪ اﻛﺜﺮ ﻣﻦ اﻟﻤﻜﺘﻤﻠﯿﻦ اﻟﻨﻤﻮ‬
- ‫اﻟﻌﻜﺲ‬
Gram + and Gram – Bacteria
116 Definition passive diffusion ?
117 If late what to do MMR Vaccine?
118 When take MCV4 Vaccine ?
119 Have a relation between MMR and autism?
120 Vitamin D, is beneficial for which Mineral ?
Calcium
121 Degree Temperature between 15-35 , which means?
Warm
122 Question about Cohort Study
123 Question about primary Resource ?
RCT
124 Disadvantage of RCT ?
125 3 question about Cytochrome? tramadol & atomoxiten & atazanavir
126 Definition of confidently?
127 Definition of Justice?
128 Definition of palliative therapy?
129 Interaction between Johns wart with SSRI , what happens ?
130 What is the meaning of Phase 3 in Cardiac Action Potential ?
131 Side effect Amiodaron ?
Pulmonary Firbosis
132 Which osmotic diuretic?
Mannitol
133 SE Lisinopril?
angioedema
134 high intensity statins?
Atorva 40
135 Gemfibrazil interaction with what?
Statin
136 Which Class is omeprazole?
137 Dual therapy for H,pylori ?
138 Vancomycin which cause ?
Red man Syndrome
139 How many drugs are for TB ?
140 Which cause hypoglycemia ?
Glipizide
141 Best place for absorption for Acidic drug ?
142 Antiplatelet cause dyspepsia ?
Ticagrlor
143 Clozapine side effect?
Agenulosystosis
144 When using Clozapine , which monitoring?
ANC
145 Side effect Haloperidol?
EPS
146 Parkinson drugs?
147 What does STEMI mean?
148 Side effect of Morphine ?
Constipation
149 Questions about Drug Class for ( diarrhea & Constipations )
150 2 question about Trastzimab & Inflexmab
151 3 question about Methtrxate
152 What is the benefit for Bentonite and boric acid in prescription ?
153 Lithium play a role for what?
Sodium
154 Question about ( Potency & Efficacy )
155 which of the followings may cause SJS?
156 Ergot alkaloids used for migraines?
A- Dihydroergometerin
B- Ergometrin
C-Dihydroergotamin
D-Ergotamin
157 Active form of vit.d for renal patient.?
Calcitirol
158 Which is correct regarding tetanus ?
A- tetanus vaccine increase activity of diphtheria vaccine
B- tetanus vaccine decrease activity of diphtheria vaccine
C- tetanus is exclusively environmental born pathogen
D- tetanus is both environmental and transmitted pathogen
159 Pt. Has vaginal infection with Candida albicans, what is the antifungal ?
Cloitrmazol
Fluconazol
160 Pt. Has surgery after 6 months , which is longest NSAID duration he can take ?
A- Ibuprofen
B- diclofenac
C- naproxen
D- piroxicam
longer half life ‫ﺷﮭﻮر‬
٦ ‫ﻋﺸﺎن ﯾﻜﻮن اﺳﮭﻞ ﻟﻠﻤﺮﯾﺾ ﯾﺎﺧﺪه ﻣﺮة واﺣﺪة ﯾﻮﻣﯿﺎ ﻟﻤﺪة‬
‫ ﺣﻄﯿﺖ‬piroxicam
161 What is the treatment for TB and for how long ?
162 Vancomycin MRSA ?
163 MSSA ?
164 Sildenafil Which Cause ?
165 Heart attack angina , Cardiology Problem !
166 Case : Patient Had Dm, HTN, and use Drug ( Gliclized / ARB / Cholestrol drug / Statin /
Duritic ) and Still have Hyperglycemia , which will do ?
167 Nitrofurantoin Side effect?
brown color of urine
168 Side effect ACEI ?
Dry Cough
169 Case : Patient use ACEI and had SE cough , which doing ?
Change drug to ARB
170 Antidote for
-Paracetamol
Morphine
Carbamzipine
Dapigatran
171 Drug-Drug interaction with Warfarin ( Drug / Food )
172 Treatment for Multiple Sclerosis ?
Fingolimod

173 Sturcutre :
Aspirin
Amphetamin
Clozapine
174

175 Questions about (Asthma , AFib, Dm , HTN )


176 ‫ “ ﺳﺆال دواء ﻣﻜﺘﺸﻒ ﺟﺪﯾﺪ ﯾﻌﻤﻞ ﯾﻜﻮرﺑﻞ ؟‬Curable “
Hepatitis A
Hepatitis B
Hepatitis C
Hepatitis D
Curable ً ‫ﺗﻤﺎﻣﺂ‬ ‫= ﯾﻌ ﻨ ﻲ ﯾﺸ ﻔ ﻲ ﻣ ﻨﮫ‬
Hepatitis C

177 Patient take aspirin as Prophylaxis for Colon Cancer and He had Bleeding , for how long use
aspirin ?
6 to 12 months
2 years
5 to 10 years.
More than 20 years
178 Acetaminophen Structure, written formula?
C8H9NO2
179 When stop using Fingolimod , for pregnancy ?
Before 2 month
6 month
Safe for Pregnancy
Before 6 months
180 Structure question about Natural prodrug for anticoagulation
181 Metronidazole can’t use for what?
Smoker patient
Drink alcohol
182 Amoxiclin ( Amoxil )

183 Amoxicillin Structure , and questions about ( Sensitivity group reaction)?


184 Treatment Malaria in Saudi Arabia ?
185 Treatment for pinworm ttt ?
Mebendazole, Albendazole, or pyrantel pamoate
186

٠ ‫ﻟﺰم ﺗﻌﺮف اﻟﻔﺮق ﺑﯿﻦ اﻷﻧﻮاع اﻟﺜﻼﺛﮫ ﻻن ﺟﺎب اﻟﻜﻼم اﻟﻠﻲ ﺗﺤﺖ ﻛﻞ ﺻﻮره و ﻗﺎل ھﺬا ﺗﺒﻊ اﯾﺶ‬
187 Sturcutre for drug use in Absent Seizure ?
Questions 6 in Calculations ?
188 Antidote for
Acetaminophen
Apixpan
BB
Codine
189 UTI ttt for pregnant and female ?
190 Which for Herbal decrease Warfarin effect?
Green Tea
191 Oral Osteoporosis drug cause Esophageal ulceration?
Alendronate, Alendronate, Resindronate
192 side effect of TB medication:
Hepatotoxicty
193 Name of B12 ?
Cyanocobalamine
194 Patient 75 years old , had atrial fibrillation , DM , HTN , Dyslipidemia , and Use several drug (
Aspirin / Atorva / Captopril / Metformin )
Questions is : Is the drugs satiable for patient ?
Answer : when calculate SHAD-Vas Score when had (2) , so : the patients should use
anticoagulation
195 act like ondansetron?
Clove
Ginger
Cinnamon
Liqurace
196 patient has Atrial fibraltion and has torsade de point? And the question want me to choose from
Class 3 anti arrhythmic?
1- Amiodarone
2- verapamil
3- defolitde
4- sotalol
197 drug recently discovered it cause DM?
Statin
198 women positive (HCG ) Human chorionic gonadotropin , what medication is appropriate for
her?
1- warfarin
2- isoretanoin
3- Tetracycline
4- Levofilaxin
199 Sunburn herbal medicine?
Aloe Vera
200 number Hep A vaccine?
2 dose
201 active moiety of fluoroquinolone ?
‫ ﺟﺎب‬structure ‫ وﻧﺤﺪد اﯾﺖ‬Active moiety
202 Recommended Vaccine for HIV patient and CD4>555?
203 ‫ﻛﺎن اﻟﺴﺆال ﻗﻄﺮة ﻣﻀﺎد ﯾﺎﺧﺬھﺎ اﻟﺒﯿﺒﻲ اول ﻣﺎ ﯾﺘﻮﻟﺪ ﻋﺸﺎن وﻗﺎﯾﺔ ﻣﻦ اي ﻧﻮع ﺑﻜﺘﯿﺮﯾﺎ؟‬
gonococal but I’m not Sure
204 patient colonized with MRSA what we will give him?
1- Intranasal Vancomycin
2- Intranasal muprocin
205 Sutucture :
Active metabolite of Diclofenac

206 septic shock patient unresponsive to Fluid what will you Give?
1- vasopressin
2- dopamine
3- ephenphrine
207 ‫ﺟﺎ ﺳﺆال اﻧﻮا ﻣﺮﯾﺾ ﯾﺘﻌﺎﻟﺞ ﻓﻲ ﻣﺴﺘﺸﻔﻰ ﺗﻌﻠﯿﻤﻲ وﻛﺎن ﯾﺤﺘﺎج‬
Bronchoscopy
chest X-ray ‫ﻗﺎم ﻗﺎل اﻟﻄﺒﯿﺐ ﺧﻠﯿﻨﺎ ﻧﻌﻤﻠﮫ‬
‫? ﻋﺸﺎن اﻟﻄﻼب ﯾﺘﻌﻠﻤﻮا‬

١-‫ﻻ ﻣﺎ ﯾﻨﻔﻊ ﻋﺸﺎن ﻣﺎ ﻧﺤﻂ اﻟﺒﯿﺸﻨﺖ ﻓﻲ ﺧﻄﺮ‬


٢-‫ﻻزم اﻟﻄﺒﯿﺐ ﯾﺸﺮف ﻋﻠﻰ اﻟﺤﺎﻟﺔ‬
208 What are the five rights?
Right patient
Right dose
Right time
Right drug
Right route
209 doxorubicin what should we monitor?
ECG because it’s cardiotoxic
210 patient has dental procedure doctor prescribed antibiotic (Augmentin), Why ?
prosthetic valve replacement
211 Antiepleptic cause Nephrolithiasis?
Zonisamide
212 Too many questions about Arrythmia , Important to study it
213 Ibuprofen dose OTC ?
1200 mg
214 athlete..accident while playing football ..inj?
Ketorlac
Not have diclofenac in Answers
215 Beriberi due to difficiency of ?
Thiamin
216 Test before clozapine ?
ANC : Absolute Neutrophils count
217 case ..pt blood pressure 70 / ?
Hypertonic
218 OU TID ?
Both eye 3 times
219 Status epileptic ?
Take phenytoin
Increase phenytoin dose
Iv lorazepam
Carbamezpine
220 Two different eye drops preparation?
5 min
221 Rare disease ?
Case control
222 Study with risk factor?
Prospective cohort study
223 Study ..prevalence?
Cross sectional
224 bioavilability defintion?
225 Resource planning?
Goals and strategy
226 Case with food allergy?
IgE
227 Case with drug allergy?
Immediate

228 case .. increase dose of oxycodone?


Constipation
229 insomnia
Zolpidem
230 traveller insomnia ?
zolpidem, Melatonin
231 metronidazole avoid with ?
Alcohol
232 case iron toxicity ?
Desferoxamine
233 case with boy take bottle of multi vitamin 2 hrs ago?
Gastric levage
234 A Girl inhaled white powder..by examination found vit K slow than usual range ?
Warfarin in options
235 incedince ?
New cases.
236 450 million of people has DM ..this called ?
Prevalence
237 Spread world wide mean?
Pandemic
238 in 1980 average age in saudi people = 60 , in 2010 = 78 ..this mean ?
Decrease mortality.
239 Alzahemier Pt with Rivestigmine ..feeling nausea vomiting?
- metoclopramide
- suppository
Don’t know answer
240 pregnant HIV ttt ?
Zidno
241 lyme disease?
Bacteria
242 Patient had allergy for pencillin , which sturucture should be avoid ?
Choose structure with “ S “
243 codien to morphine ?

244 calcium dose in pregnancy?


1200-1500
245 warfarin ?
Take constant level of green leaves
246 anti coagulant cause dyspepsia?
Dabigatren
247 pt with DVT ..duration for ttt by heparin ?
3 months
248 heparin induce thrombocytopenia use ?
Argatoban
249 Norepinephrine is ?
1-Hormone and enzyme
2-Coenzyme and neurotransmitter
3- Hormone and neurotransmitter
250 Atropine belladonna..name due to ?
plant
251 Alkaloid cause hepatotoxicity ?
‫“ ﻣﻦ اﻟﺨﯿﺎرات ﻓﯿﮫ‬tropane”
252 plasma volume for man 70 kg ?
3.5 ‫ﻛﺎن ﻓﯿﮫ اﺧﺘﯿﺎرات اﺧﺘﺮت‬
253 Clopidogrel increase efficacy if taking with ? Chooses
Omeprazole,
amiodarone
erythromycin
phynotin
254 trasruzmab MOA ?
HER2 inhibitor
255 Bentonite ?
Effective emulsifying
256 Lactose with hepatic encephalopathy moa ?
Acidfy colon

257 produced in liver and stored in gallbladder?


258 IG in colostrum?
‫ ﻣﻜﻨﺶ ﻓﯿﮫ‬A
‫وﻛﺎن ﺟﺎﯾﺐ ﻛﻞ اﺗﻨﯿﻦ ﻣﻊ ﺑﻌﺾ‬
‫ اﺧﺘﺮت‬G,M

259 hypothyroidism..hormones ?
TSH high , thyroid hormone low .
260 Case : High TSH , low T3 , which Treatment ?
Leveothyroxine
261 case ..pt hyperkalemia ?
spironlactone ,
lisnopril ,
insulin
‫ ﻛﺎن ال‬insulin ‫ اﻟﺤﺎﺟﺔ اﻟﻮﺣﯿﺪة ﻓﯿﮭﻢ اﻟﻠﻲ ﺑﺘﻌﻤﻞ‬hypokalemia
262 case ..pt DM symptoms....wt lose ..HTN ,heart failure ..drug choice for DM ?
Insulin
liraglutide
metformin
Glypuride
263 Deep wound + unknown history ?
Td+ TIG
264 Minor wound + unknown history?
TD
265 live vaccine?
MMR

266 herpes simplex ..vaccine?


Vareicelal
Zoster
267 Dental :
Streptococcus mutans .
268 Avoided in nursing mother :
Aspirin
269 Contraindicated in breastfeeding:
Lithium
270 insulin take single dose in day :
Glargine
271 Asthma ..exircise :
salmetrol
corticosteroid inhalation,
ipratipum

NO “ SABA “ in Choosse
272 Intermitten asthma :
273 18 month boy with cold symptoms :
Fluids
274 old pt with depression :
All options TCA with duloxtine
275 Atenolol MOA :
Selective B blocker
276 antibiotic used with surgical prophylaxis?
277 women with acute migraine?
Triptan
ibuprofen
paracetamol
278 Adverse drug reactions..reported to ?
SFDa
279 Role of therapeutics and drug committee?
Drug formualy
280 Who is responsible for immunity (vaccination) in saudi ?
MOH
281 2 drugs equivalent..we will choose one of them according to ?
Cost minimization.
282 2 drugs ..we wanna choose one that decrease mortality of cancer ?
Cost effectiveness or cost utility
283 drug interact with food and antacid ?
Doxycycline
284 case ..baby ..choose suitable antibiotic ?
Flouroquinolone
tetracycline
TMP
amoxicillin/ clavunic acid
285 drug causes brown urine color ?
286 vancomycin trough ?
before 4th dose
287 Vancomycin trough level ? after any dose ?
3th dose
288 which should be do before give vancomycin!
‫ اﺧﺘﺮت‬renal insufficient
289 postural hypotension
Prazocin
290 case ..pt with HTN take ACE ..suffer from coughing ?
‫وﻛﺎن اﻧﺴﺐ ﺣﺎﺟﺔ ﻓﻲ اﻻﺧﺘﯿﺎرات أﻧﮫ ﯾﺎﺧﺪ دوا ﻛﺤﺔ‬
291 case ..women with albicans bacteria?
Ttt ..fucanzole
292 case pt ..DM ..take insulin and others drugs..refused to take insulin ..he said he cause wt gain
‫ ﻣﺮة ﺟﺎوﺑﺘﮫ ﺑﺄن ﺣﺎﻟﺘﮫ ﻻزم ﺗﺎﺧﺪ‬.. ‫ دي ﺟﺖ ﻣﺮﺗﯿﻦ ﺑﻄﺮﯾﻘﺘﯿﻦ ﻣﺨﺘﻠﻔﯿﻦ‬insulin ‫ و ﻣﯿﻨﻔﻌﺶ ﯾﺒﺪﻟﮫ ﺑﺤﺎﺟﺔ ﺗﺎﻧﯿﺔ‬.
‫ﺑﺄھﻤﯿﺔ اﻻﻧﺴﻮﻟﯿﻦ ﻟﺤﺎﻟﺘﮫ‬.. ‫ وﻣﺮة ﺟﺎوﺑﺘﮫ‬.
293 ‫ﻛﺘﯿﺮ وﻛﺪا‬.. ‫! ﻣﺮﯾﺾ ﺟﯿﻠﻚ ﯾﺼﺮف روﺷﺘﺔ وﺑﯿﺸﺘﻜﻲ ان دي ﺗﺎﻧﻲ ﻣﺮة ﯾﻜﺸﻒ ﺧﻼل ﻓﺘﺮة ﻗﺼﯿﺮة وف ﻛﻞ ﻣﺮة ﺑﯿﺪﻓﻊ ﻓﻠﻮس‬
‫ وﻻ ﺗﻘﻮﻟﮫ أﻧﮫ اﻛﯿﺪ اﻟﺪﻛﺘﻮر ﺷﺎف ان ﺣﺎﻟﺘﻚ ﺗﺴﺘﺪﻋﻲ ﻛﺪا‬،‫ وﻻ ﻣﺘﺼﺮﻓﺶ اﻟﺪوا‬،‫اﻧﻚ ﺗﻘﻠﻠﮫ اﻟﺠﺮﻋﺔ‬.. ‫ﻛﺎن ﻣﻦ ﺿﻤﻦ اﻻﺧﺘﯿﺎرات‬
294 ‫ ﻣﯿﻦ اﻟﻤﺮﯾﺾ اﻟﻠﻲ ﻻزم ﻧﺎﺟﻠﮫ ال‬vaccinatin :
Patient just finished chemotherapy cycle
295 case ..women ..DM symptoms..obese ?
Meformin
296 case ..pt take 850 mg metformin .. 5.5, 6,7, 6.5 ‫ ﻗﯿﺎﺳﺎت ﻛﺎﻧﺖ‬٤ ‫وﺟﺎﯾﺐ آﺧﺮ‬
Increase dose metformin
297 ‫ده اﺳﻤﮫ اﯾﮫ ؟‬.. ‫واﻧﺖ ﺳﻮاء ھﺘﻨﻔﺬ رﻏﺒﺘﮫ او ﻻ‬..‫ﻣﺮﯾﺾ ﺑﻤﺮض ﻣﺰﻣﻦ وﻗﺮر اﻧﮫ ﻣﺶ ﻋﺎﯾﺰﯾﺎﺧﺪ دواء ﺗﺎﻧﻲ‬
velicity, fidelity
Autonomy
298 ‫وزوﺟﺘﮫ ﻓﻲ اﻟﻐﺮﻓﺔ اﻟﻠﻲ ﺟﻨﺒﮫ ﺑﺘﻔﮭﻢ ﻋﺮﺑﻲ ؟‬.. ‫ﻣﺶ ﻗﺎدر ﯾﻔﮭﻢ ﻣﻨﻚ ﺣﺎﺟﺔ‬.. ‫ﻣﺮﯾﺾ ھﻨﺪي‬
‫ھﺘﺠﯿﺒﮭﺎ ﺗﺘﺮﺟﻢ اﻟﻠﻲ ﺑﺘﻘﻮﻟﮫ‬
299 Isomer sturuce ?
300 Epimer structure ?
301 Dish ?
Hypertension
302 What is the meaning of GABA, HMG-CO A abbreviation?
303 Medication That if given by error , Can cause Serious injury ?
-look Alike / Sound Alike
-High risk medication
304 Side effect of Niacin ?
Flushing
305 Amphetamine MoA?

306 drug need Liver monitoring


Methotrexate
Adalimumab
307 Drug Couse teeth descolroization
Tetracycline
308 Case with pneumonia and in 3 days symptoms resolve
What is the medical intervention?
Stop all drugs
Give oral cef and oral azithromycin
Give IV cef and IV azithromycin
Oral cef and doxycyline
309 Women being treated for ----- and feels like fainting when she stands up what is the drug causing
it?
_ Bromocriptin

310 Patient complaining of pain, diagnosed with sickle cell, what drug to use?
What are Fungi proprieties
311 Pseudo antibiotic treatment
312 Chemical drug resonsible of gas formation in new born babies?
313 Women has signs of vitamin A deficiency, what are its natural forms?
Olive oil
Liver oil
Argan oil
314 Time of vaccine Heptatits A
315 Type of resource ( college of pharmacyactivity about diabetes)?
Tertiary
316 Score of atrial fibrillation?

317 Antidote of digoxin?

318 Patient travels to a city with a disease which types of vaccine to take?
Heptitis A or with immunoglobulin?

319 Mechanism of cipro?

320 Case… Give folate to which drug?

321 Bacteria catalase which types?

322 Validity definition?

323 Cost benefit anaylysis?


Monetary untis
324 Cost effectiviness anaylysis?
Natural units
325 Cost utility analysis?
QALY
326 Ig in asthma?
IgE
327 Calculation:
Infusion rate and rate of flow
LD
Calculate paracetamol dose for child
‫ ﺑﺲ ﻣﺎدة ﺛﺎﻧﯿﮫ ﻧﺴﯿﺘﮭﺎ‬٪٠.٩ ‫وﻣﻌﺎدﻟﺔ ﻛﺎﺗﺐ اﺳﻢ اﻟﻤﺮﻛﺐ ﻛﯿﻤﺎﺋﻲ واﻧﺘﻲ ﺗﻌﺮﻓﯿﻦ وش ﺗﺮﻛﯿﺰه وﺗﻌﻮﺿﯿﻦ زي اﻟﺼﻮدﯾﻮم ﻛﻠﻮراﯾﺪ‬
Which anticoagulant need parental administration before oral administration?
Dabigatran , rivaroxaban , abixaban , ‫ودواء راﺑﻊ ﻧﺴﯿﺘﮫ‬
Warfain , Minimum 5 day
Meaning Question is : drug need Bridging ,
jet lag treatment ?
Melatonin
Two different eye drop ?
Wait 5 minut before add second
Which Anticoagulation Direct – thrombin inhibt ?
Dabigatrin
Argatroban
Both are coorrect
Which meaning 800 ?
Hazard drug

‫اﻻ ﺳﺆال واﺣﺪ وﷲ ﺗﺬﻛﺮﺗﮫ وش ﻟﻮن اﻟﺒﻮﻛﺲ ﺣﻖ اﻟﻨﻔﺎﯾﺎت‬


Yellow
Sturcutre for Parbitorate , and Select the Short acting ? “ without Name , only Structure “
Short acting is :
Hexobarbitone
Pentobarbitone
Secobarbital
Amobarbital
‫اﺑﺤﺜﻮا ﻋﻦ ﺳﺘﺮﻛﺸﺮز ھﺎﻷدوﯾﮫ‬
Aspirin Structure?
Is Covalent bond is responsible for antiplatelt
Isomer
Ipemer
For acyclover to des..lover

Patient had paint and GI bleeding , I think had HTN , DM . which analgesic will gave ?
Ibuprofin
Methadon ?
Hydro codon
Analgesic for patient using warfarin and have heart disease ?
Statine give with ?
Co-enzym
plasmin in bacteria responsible for ?
Drug for Lice ?
When drug A increase a Therapeutic action of D B ?
-increase metabolite
-decrease metabolite
For travel insomnia ?
Patient use warfarin and SGLT2 inhibitors drug , which is serious/ danager will happen ?
Liver function
weight
Case for patient , had COPD ‫وﯾﻌﻄﯿﻚ ﺻﻔﺎﺗﮫ‬
Q : which COPD Category for the patient ?
Pharmacophore definition?
Pharmacovigilance definition ?
Pharmacosoioly definition ?
Counseling for Asthmatic Patient ?
-reduce weight
-
-
Cases / Ashmatic patient and obesity what management for their thereby ?
1-dose of aldoronate in postmenopausal ?
2-Anticogulant safe pregnant?
3-clincal trial to approve drug ?
3-MRSA tratment ?
4-Aminoglycosid affect any organ montring ?
4-drug increase warfaring ? ‫ﻛﻞ اﻟﺨﯿﺎرات ﻛﺎﻧﺖ ﻣﻦ اﻻﻧﺘﻲ ﺑﯿﻮﺗﻚ‬
5- benzodizpam MOA?
6- aspirin pka 3.2 absorb in acidic ?6-54-3
7-mechinal valal which drag anticogulant ?
8-streptococcus and absence O2?
9-s.j and serotenin ?
11-pandemic ? Prevlence ?
12- ondasetron like ?
13-mRNa?
Transcripition –
Translation –
Transportion
Transcriptase if RNA to DNA
14-Vaccin ..... autism ?
15-Live viccination ?
16- ‫ اﻋﻄﺎه ﻓﯿﺘﺎﻣﯿﻦ‬k..... ‫ وﻗﺎﻟﻲ ﻛﻢ‬INR ?
17- drug antibotitc time depanted ?
18-drug antibiotic conc depanted ?
19- lamotrigine side effect ?
20- niacin side effect ?
21- B12 ‫? ﻧﻘﺼﺎﻧﮫ ﯾﺴﺒﺐ‬
22-‫ ﺷﻮﻓﻮا اﻧﻮاع ﻛﻠﮭﺎ‬.. ‫ﺟﺎب ﻛﯿﺴﺲ وﻗﺎﻟﻲ اﯾﺶ ﻧﻮع اﻻﻧﯿﻤﯿﺎ ﻋﻨﺪدو‬
23- ‫ ﻛﯿﻒ ﺗﺘﺠﻨﺒﻲ‬error medication ?
24- codine enzyme to morphine?
‫ اﻻﺧﺘﺼﺎرات ﺟﺎﺑﻮھﺎ ﻛﺜﯿﺮ زي‬o.u ‫رﻛﺰو ﻋﻠﻰ اﻟﻌﯿﻦ واﻻذن‬
‫ﻛﯿﺲ ﺟﺎت واﺣﺪ ﺟﺎ ﯾﺸﺘﻜﻲ اﻟﻢ ﻓﻲ ﻧﺎﺣﯿﮫ اﻟﯿﻤﯿﻦ اﯾﺶ ﺗﺘﻮﻗﻌﻲ ﻋﻨﺪه وﻣﺎﺟﺎب وﻻ ﺗﺤﻠﯿﻞ وﻻ اﻋﺮاض ؟‬
‫ھﺬي ﻛﺎﻧﺖ ﺧﯿﺎراﺗﮫ‬
Liver
Cholestasis
pancreatitis
? Patient with Ostoprosis which need for Diagnosis
X-ray –
MRI –
CT
‫ وﺟﺎ ﻛﻤﺎن ﺻﻮره ﻗﻠﺐ وﻛﺎﺗﺐ ﻋﻠﯿﮭﺎ ﺑﯿﺎﻧﺎت اﻟﻘﻠﺐ وﺟﻤﺒﮭﺎ ﻣﺨﻄﻂ اﻟﻘﻠﺐ وﻛﺎﺗﺐ اﺑﻐﺎ‬PR

When take Omeprazole ?


-Monring with meal
-Night before sleep
-As need
- stimulate adrenergic
‫ اﻟﻐﺪد اﻟﺪرﻗﯿﮫ وﺗﻘﺮﯾﺒﺎ اﻏﻠﺐ اﻻﻣﺮاض ﯾﺒﻐﻰ ﻓﯿﺘﺎﻣﯿﻦ ﯾﺘﺎﺧﺬ ﻣﻌﺎھﺎ ﻛﻢ ﻛﯿﺲ ﺟﺎء ﻣﻨﮭﺎ‬- ‫ اﻟﻀﻐﻂ‬- ‫اﻧﻲ اﻣﺮاض اﻟﺴﻜﺮ‬
Women plan for Pregnance and she use methimazole ?
- ‫ ﺗﺎﺟﻞ اﻟﺤﻤﻞ ﻟﻐﺎﯾﮫ ﻣﺎﺗﺘﻌﺎﻓﻰ‬- ‫ ﻧﻮﻗﻔﮫ ﻟﮭﺎ ﺑﺎﻟﺘﺪرﯾﺞ وھﻮا اﻣﻦ ﻓﻲ اول ﺷﮭﻮر ﺣﻤﻠﮭﺎ او‬- ‫اﻟﺨﯿﺎرات ﻛﺎﻧﺖ ﺗﻮﻗﻒ اﻟﺪواء اول ﻣﺎﺗﺤﻤﻞ او‬
‫وﻧﺴﯿﺖ اﻟﺮاﺑﻊ ﺑﺲ ﻣﻊ دواء ﺗﺎﻧﻲ‬
‫ ﺟﺎء ﻛﻤﺎن ﺳﻮال ﻋﻦ‬Athletes ‫? اﯾﺶ اﻋﻄﯿﮫ‬
Pseudo antibiotic treatment
Lamotrigine side effect ?
Not “ Rash “ in option
amphetamine MoA ?
I think increase noradrenaline
*drug need Liver monitoring ?
Methotrexate
Adalimumab
For ECG :
*QRS action
*TV action
*Drug for control rythym
*Antiboiotoc contraindicated in pregnant 🤰
*Drug Couse teeth descolroization
Tetracycline
Case with pneumonia and in 3 days symptoms remove:
What the medical intervention
-stop all drugs
-Give oral Cef .. and oral azithromycin
-Give IV Cef + IV azithromycin
-oral cef doxycycline
Case with status epilecy , Take phenytoin
_increase phenyton dose
_iv lorazepam
_carbaamezapine
Patient with Multiple sclerosis disease, and feel dizziness with stand up, which drug should be take?

Bromocriptin
Patient Suffer from pain form Sikle cell disease , which drug should take ?
Morphin
Fungi proprieties ?
‫ﻣﺮﻛﺐ ﻛﯿﻤﺎﺋﻲ ﻣﺴﺆول ﻋﻦ ﺗﻜﻮن اﻟﻐﺎزات ﻋﻨﺪ اﻟﻤﻮاﻟﯿﺪ‬
Patient have defiiecny frim Vitamin A , when should be take Vitamin A from Natural ?
olive oil
_liver oil
Organ oil
Herbal treatment for Vomiting ?
Clove
Ginger
Cinnamon
Antidote for apixapan?
andexent
Type of resource (collage of pharmacy activity about Diabetes
Tertiary
Times of Vac Hip A?
2
Pelletierine traditional use in what?
2- ‫?? ﻋﻠﻰ أي اﺳﺎس ﯾﺘﻢ ﺗﺴﻤﯿﺔ اﻻدوﯾﺔ اﻟﻠﻲ ﺗﻜﻮن ﻣﻦ ﻧﺒﺎﺗﺎت طﺒﯿﻌﯿﮫ ا‬

‫و ﻋﻠﻰ اﺳﻢ اﻟﻌﺎﻟﻢ اﻟﻠﻲ اﻛﺘﺸﻔﮭﺎ‬


‫او اﻻﻛﺘﯿﻒ ﻛﻮﻧﺴﺘﯿﺘﻮﻧﺖ‬

3-‫ﻣﺮﯾﺾ ﻋﻨﺪه ﺣﺴﺎﺳﯿﺔ ﻣﻦ اﻟﺒﻨﺴﻠﯿﻦ و اﻻﺧﺘﯿﺎرات ﺳﺘﺮﻛﺸﺮز و اﻟﺠﻮاب ﻛﺎن ﻋﻨﺪي ﺳﺘﺮﻛﺸﺮ اﻟﻜﻮﯾﻨﻮﻟﻮن‬

Q: Amphotericin B Stucture , and Ask about What it’s ? Hydrolysis or Oxidation


Answer : ( Hydrolysis )|

Q: Same questions about Warfarin?

(conjugation)
doxorubicin class ?
6- cyclophosphamide Side effect ?
Too many question about Cancer Disease ( Use / Sied effect / MOA )
7- G+ve and G-ve Different in layers and components
8- cimetidine dicreased metabolism of drug X
9- which intracellular and extracellular elctrolyte
10- amylase and lipase Responsibilities
11- aspirin ?
‫رواﺑﻂ ﺗﺴﺎھﻤﯿﺔ‬
12- PR interval and ST segment in ECG
13- anti arrhythmia classes
‫اش اﻟﺨﻠﯿﺔ اﻟﻠﻲ ﻋﻠﻰ ﺳﻄﺤﮭﺎ اﻻﻧﺘﯿﺒﻮدي‬
T helper cell > CD4 +
IF any one give the sample for pharmacis ,what the pharmacist should do ?
patients refuse to use insulin due to weight gain ‫اش اﻟﺘﺼﺮف‬
‫ﺻﻮره اﻟﻠﯿﺜﺎل دوز و ﯾﻘﻠﻲ ﻛﻢ ﺗﺴﺎوي‬
potency and efficacy pic
20- ‫ ﺳﻮال ﯾﻘﻠﻚ‬HCG + ‫ﻓﻤﻌﻨﺎﺗﮫ اﻟﻤﺮﯾﺾ ﺣﺎﻣﻞ ﻣﻦ دون ﻣﺎ ﯾﻘﻠﻚ ھﻮ ﻣﯿﻞ او ﻓﯿﻤﯿﻞ ﺑﺲ ﺧﻼص داﻣﮫ اﯾﺠﺎﺑﻲ ﻛﻌﻨﺎﺗﮫ ﻧﺪور اﻻدوﯾﺔ‬
‫اﻟﺴﯿﻒ ﻟﻠﺤﺎﻣﻞ‬
21- UGT1A1 enzyme for which drug ?
22- enoxaparin doses
23- definitions of management
26- candida albican ttt
Calculation :
Correct Calcium
Correct Phenytoin
LD –
CrCl –
displasment

Bioavailability difinitions
fraction (%) of an administered drug that reaches the systemic circulation.
bioavailability calculation
AUC (oral)/AUC iv X 100
specific gravity
Mass / volum
calculation 10%W/W ,V/V
cost minimization analysis
disease or illness
direct medical
*Direct nonmedical cost
*non direct cost
lost productivity for patient
non direct cost
o.u t.i.d
Both eyes three times a day
a.c
Befor meal
cimetidine
decreas excreation
decreas metabolism
gentamycin
liver _adptive tissue , systemic circulation
* adptive tissue _liver _ systemic circulation
antidote iron
Defroxamine
Antidote digoxin
NORMAL range phenotin
10-20
Quinlon ___ Quinidine antiarrhthmias
veracity

* etical principle
ferros does
100 tid
‫ ﺣﺒﺎت وﺟﺊ ﺑﻌﺪھﺎ ﻣﻌﺼﺐ ﯾﺒﻐﻰ اﻛﺜﺮ واﻟﺪواء ﻣﻮ ﻣﺼﺮح ﻟﻠﺼﺮف ﻏﯿﺮ ﺛﻼث ﺣﺒﺎت‬3 ‫ﺟﺎﻧﻲ ﺷﺨﺺ ﺗﻢ ﺻﺮف ﻟﮫ ﻣﻦ ادوﯾﮫ ﻧﺎرﻛﻮﺗﻚ‬
‫واﻻﺟﻮﺑﮫ‬
‫وﺗﻄﻠﻌﮫ ﻣﻦ اﻟﺼﯿﺪﻟﯿﮫ‬
‫ﺗﺪاﻓﻊ ﻋﻦ ﻧﻔﺴﻚ‬
‫ﺗﻄﻠﺐ ﻣﻦ ﺳﻜﯿﺮﺗﻲ ﯾﻄﻠﻌﻮه‬
‫اوه ﺗﺴﻤﻊ ﻟﮫ وﺑﻌﺪﯾﻦ ﺗﻌﻠﻤﮫ ﺳﯿﺎﺳﺎت‬
‫ﺷﺨﺺ ﺷﻜﺮ واﺣﺪ ﺑﻌﺪ اﻟﻤﯿﺘﻨﻖ واﻻﻧﺘﺮﻓﯿﻮ‬
Verbal
Non verbal
warm plase
20-25
diabetic vaccination
Pneumococcal
person Hepatitis A vaccine of endmic plase
HAV+HIM
folic acid
1 month before pregnancy

om salah )‫ﺗﺒﯿﻊ ﻣﻨﺘﺠﺎت ﻋﺸﺒﯿﮫ وﻋﻤﯿﻞ اﺷﺘﻜﻰ ﻋﻠﯿﮭﺎ‬


‫ﻟﺘﺮﺧﯿﺺ اﻟﻤﻨﺘﺠﺎت ﻋﻨﺪھﺎ‬# ‫ﺗﻌﻤﻞ ﻟﯿﮭﺎ ﺗﺼﺮﯾﺦ‬
‫ﺗﻔﺘﯿﺶ‬#
TSH 👆L3👇
Levothyroxin
atomoxcin cyp450
disease of niacin
Phellegra
‫ﺷﺨﺺ ﺟﺎي وطﺎﻟﺐ دواء ﺣﻖ‬Premature ejaculation
‫ﺗﻜﻠﻤﯿﮫ ﻋﺎدي ﻗﺪام ﻧﺎس‬
‫ﻟﻮ ﺗﺼﺮﻓﯿﮫ ﻋﺎدي‬
‫ﻟﻮ ﺗﻜﺎﺑﻲ رﺷﺘﮫ وي اﻟﻌﻠﺒﮫ‬
‫ﻟﻮ ﺗﺎﺧﺪﯾﮫ ﻣﻨﻄﻘﮫ ﺧﺎﺻﮫ وﺗﺴﺄﻟﯿﮫ‬
Vit definite cause hair loss
Biotin
Iso 7

-10
-100
-1000
-10000
‫ﺟﺎﻧﻲ ﺳﺆال وﺣﺪه ﻋﻨﺪھﺎ‬hypothyroidism ‫ﺟﺎت ﺗﺎﺧﺬ‬levothyroxine ‫واﺳﺘﺸﺎرت اﻟﻔﺎرﻣﺴﺴﺖ ﻋﻦ اﻋﺸﺎب ﺗﺎﺧﺬھﺎ ﺑﺪل اﻟﻌﻼج‬
‫وش اﻟﻤﻔﺮوض ﯾﻜﻮﻧﻦ اﻟﺮد اﻟﺼﺢ‬
‫ اﻻﻋﺸﺎب ﻣﺎﻟﮫ ﻣﻮاد ﻛﯿﻤﯿﺎﺋﯿﮫ ﻓﻌّﺎﻟﮫ ﻣﺜﻞ اﻻدوﯾﮫ‬-
‫ اﻻﻋﺸﺎب ﻣﻮ ﻣﻌﺮوف ﻣﻜﻮﻧﺎﺗﮭﺎ‬-
‫ اﻻﻋﺸﺎب ﺗﺘﻌﺎرض‬-
drug mask hypoglycemia?
B blocker
Nurse received extra vial in the bag what should we do ?
Return to pharmacy
high intensity statins?
Atrov 40
Ergot alkaloids used for migraines
Dihydroergotamin
When patient use clozapine what monitoring?
“ agranulocytosis “ ‫ﺗﺎﻛﺪو‬
type of WBC cause antibody production
Lymphocytes
which of the following not essential amino acid
Arginine
what is warfarin level in mechanical valve ?
2.5 – 3.5
Patient with HIV what monitoring?
CD4 “ ‫ﺗﺎﻛﺪو‬
Cost effectiveness?
To identify, examine, and compare relevant costs and consequences (clinical outcomes) of competing
drug regimens and interventions.
‫ دﯾﺴﻤﺒﺮ ﻣﺘﻰ اﺧﺮ ﻣﺮه ﯾﻨﻔﻊ ﻧﺎﺧﺬه؟‬31 ‫دواء ﺑﯿﻨﺘﮭﻲ ﺗﺎرﯾﺨﮫ ﯾﻮم‬
‫ اﻛﺘﻮﺑﺮ‬1 ، ‫ دﯾﺴﻤﺒﺮ‬1 ،‫دﯾﺴﻤﺒﺮ‬31
septic shock
Dopamine , norepinephrine, epinephrine
Patient with hypothyroidism and she becomes pregnant what use dose
20
25
100
125
50
normal creatinine level for male

What mean MMR


Measles , mumps , rubella
How can protect stomach when use metformen
Take with antacid
After meal
Between meals
Anti-dote cocaine

How many calories in one gram


10
What’s the immunoglobulin in MALT ?
igA
How is frist one to prevent medaction error?!
Pharmacist, patien , nurses
what is ingredient use to prepare suppository?
Cocoa butter
vitC used for
Preservative
Vitc diafactinsy?!
Scurvy
car driver wants antihistamine
Loratidine
Time for drawn gentamicin?
30 minutes before 4th or 3rd dose
Women has signs of vitamin A deficiency, what are its natural forms?
Olive oil
Liver oil
Argan oil
Health care in pharmacy work and have infaction how the infaction give to patient?
Hand
Air

live vaccine?
MMR
Avoided in nursing mother :
Insline
Case ..pt DM 2symptoms....whent to lose wieght..HTN ,heart failure ..do life style chang and excirses
drug choice for DM 2?
Insulin
liraglutide
✅metformin
Glypuride
Case ..pt DM symptoms....wt lose ..HTN ,heart failure ..drug choice for DM to be wight gain?
Insulin
Clopidogrel increase efficacy if taking with Omeprazole?!
Cyp c19
women positive (HCG ) Human chorionic gonadotropin , what medication is appropriate for her?
1- warfarin
2- isoretanoin
3- Tetracycline 4- ✅Levofilaxin
Which vit use for vomting in pregnncy?
B6
Hf and Atrial fibraltion ?
Amidorone , dofetolide
Treatment Malaria in Saudi Arabia ?

Name vit b12


Cyanocobalamine
When stop using Fingolimod , for pregnancy ?
2 month
Metronidazole can’t use for what?
Smoker patient
Drink alcohol✔
Depression
Cardoovaculer deasise
Plant used to treat vomiting?
Ginger
Child one years ago, had immunodeficiency, he took before 4 month IVIg, and now it’s time to take
MMR Vaccine
Give vaccine
B- don’t give because he took IVIG
C- ✅don’t give because he has immunodeficiency
Montoring of lithume
Cbc
Liver fanction
Renel fanction
How many phases of drug study until it’s approved?
3 phases
Pt come to hosptial with lab result AST is high ,ALT high and G high amylase is normal~* bilirubin ia
normal*~
A-*✅heptic cirrhosis*
B-heptic stenosis
C-pancriatitis
Bilirubin normal (liver cirrosis)
Bilirubin is high(liver stenosis)
High enzyme malyse(pancriatitis)

Pseudomonas aeruginosa in gram stain ?


✅gram negative bacilli
what is warfarin level in mechanical valve ?
2.5-3.5
women 23 years old has UTI what is best option for her ?
Trimethoprin-sulfamethoxazole 160/800 mg BID for 3 days
Pt have shotage breath and go to emrgancy and have fever and cute cough .what digonose of pt?
respiratory syncytial virus‫ﺗﺎﻛﺪو‬
‫ﻛﯿﺲ وﺳﺄل ﻋﻦ ﻣﻌﻨﻰ‬P wave= Atrium depolarization
QRS complex= Ventricles depolarization T wave= Ventricles Repolarization
Drug C.I in sulfa allergy?
Bactrim SMX/TMP and other medications that contain sulfa

Affinity Defenation?
What the reseson for deficancy hematocrit Vitamin B12 or ferros sulfate
Struturs about pencillin and trizole?
Whats antidote for bzd?
Antiarrhythmic and hr control?

🔆‫ ﺻﺢ اﻣﺲ ﻛﻤﺎن ﺟﺎﻧﻲ ﺳﺆال ﻋﻦ‬decongestant ‫اﻟﻰ ﻋﻤﺮ ﻛﻢ ﻣﻤﻨﻮع ﻓﻲ اﻷطﻔﺎل‬


What the reason for Raynaud’s Disease ?

A person with Raynaud’s disease experiences pain in the extremities, for example, the fingers, when
temperatures drop.
How to prevent recurrent of UTI fore female?
C eprofloxacin
Nitrofurination
Ceftaxon
Metronidazole
What the treatment for for Raynaud’s Disease ?

Inhaler have 200 puff , and each puff contain 0.135 ml , and Concentration ….
Question is :calculate the concentration for one puff ?

‫ ﯾﻌﻨﻲ ﻛﻢ ﺗﺤﺘﻮي‬gram ?
‫طﺮﻓﯿﻦ ف وﺳﻄﯿﻦ ﯾﻌﻨﻲ‬
Cross sealing definition
‫ ﺳﻨﻮات‬٦ ‫ھﯿﺌﺔ اﻟﻐﺬاء و اﻟﺪواء ﻣﻨﻌﺖ اﺳﺘﺨﺪام ادوﯾﺔ اﻟﻜﺤﺔ ﻻﻗﻞ ﻣﻦ‬
Zonisamide induce kidney stones
Q: Encapsulated
1- E.coli
2- strept pneumonia

Q:What’s the immunoglobulin in MALT ?


IgA

Q:How is frist one to prevent medaction error?!


Pharmacist
Naurse
Doctor
Patient

Q: what is ingredient use to prepare suppository?


Cocoa butter
Q:Pregnant women has hepatitis B when she delivers we must give her baby ?
Hepatitis B vaccine + Hepatitis B immunoglobulins

Q: vit Cused for?


Preservative
Q: Vit C diafactinsy?!
Sarvay

Q:Car driver wants antihistamine ? Lortidine


Nurse received extra vial in the bag what should we do ?
Return to pharmacy
Time for drawn gentamicin?
30 minutes before 4th or 3rd dose(‫)ﻛﺮره ﻣﺮﺗﯿﻦ ﻣﺮه ﺳﻮال ﻋﺎدي وﻣﺮه ﻛﯿﺲ‬
Enoxaparin 80 sc to time dily ‫ﻛﻢ ﺗﻌﻄﯿﮫ ﻣﻦ ﻻﻛﺴﯿﻨﻮ‬٨٠ ‫ﻛﯿﻒ ﺑﺘﻜﻮن ﻣﻜﺘﻮﺑﮫ ﺑﺎﻻوردر وﺟﺎب ﻧﻔﺴﮭﺎ ﺑﺲ ﻛﯿﺲ ان ﺑﯿﺸﻨﺖ وزﻧﮫ‬
‫ﺑﺮﯾﻦ‬
Drug C.I in sulfa allergy?
Bactrim SMX/TMP and other medications that contain sulfa
(Pt have shotage breath and go to emrgancy and have fever and cute cough .what digonose of
pt?respiratory syncytial virus‫ﺟﺎب ﻛﯿﺲ‬
P wave= Atrium depolarization
QRS complex= Ventricles depolarization T wave= Ventricles Repolarization
women 23 years old has UTI what is best option for her ?
Trimethoprin-sulfamethoxazole 160/800 mg BID for 3 days
what is warfarin level in mechanical valve ?
2.5 to 3.5
Pseudomonas aeruginosa in gram stain ? A - gram positive cocci
B- gram nigative cocci
C- gram positive bacilli
D- ✅gram negative bacilli
١-١٠٠٠ ‫ھﺬا ﺳﻮال ﺑﺘﺠﻤﯿﻌﺎت‬
‫ﺣﺼﮫ‬.‫وﺻﺢ ﻛﻼم د‬
Pt come to hosptial with lab result AST is high ,ALT high and G high amylase is normal~* bilirubin ia
normal*~
A-✅heptic cirrhosis
B-heptic stenosis
C-pancriatitis
Bilirubin normal (liver cirrosis)
Bilirubin is high(liver stenosis)
High enzyme malyse(pancriatitis)
How many phases of drug study until it’s approved? 1 phase
2 phases
✅3 phases
4 phases
Plant used to treat vomiting?
✅Ginger
Cinnamon
Clove

Child one years ago, had immunodeficiency, he took before 4 month IVIg, and now it’s time to take
MMR Vaccine A -Give vaccine
B- don’t give because he took IVIG
C- ✅don’t give because he has immunodeficiency
high intensity statins?
Atorva 40
Montoring of lithume
Cbc
Liver fanction
Renel fanction
When stop using Fingolimod , for pregnancy ? ✅Before 2 month
6 month
Safe for Pregnancy
Before 6 months
Metronidazole can’t use for what?
Smoker patient
Drink alcohol
Depression
Cardoovaculer deasise
Treatment Malaria in Saudi Arabia ?
Name of B12 ?
Cyanocobalamine
Which vit use for vomting in pregnncy? Vit 6
Hf and Atrial fibraltion ?
1- ✅Amiodarone
2- verapamil
4- sotalol
women positive (HCG ) Human chorionic gonadotropin , what medication is appropriate for her?
1- warfarin
2- isoretanoin
3- Tetracycline 4- ✅Levofilaxin
Clopidogrel increase efficacy if taking with Omeprazole?! Cyp c19
Case ..pt DM symptoms....wt lose ..HTN ,heart failure ..drug choice for DM to be wight gain?
✅Insulin
liraglutide
metformin
Glypuride
Case ..pt DM 2symptoms....whent to lose wieght..HTN ,heart failure ..do life style chang and excirses
drug choice for DM 2? Insulin
liraglutide
✅metformin
Glypuride
live vaccine?
MMR
Avoided in nursing mother :
Insline
Health care in pharmacy work and have infaction how the infaction give to patient?
Hand
Aire
‫ﻣﺎذﻛﺮ اﻟﺒﺎﻗﻲ‬
Women has signs of vitamin A deficiency, what are its natural forms?
Olive oil
Liver oil
Argan oil
if prescription from Emergency
Q- Which of the following set the rules and regulations for pharmacy practice in Saudi Arabia?
A. SFDA
B. ✅MOH
C. SCFHS
D. Saudi Pharmaceutical Association
Q- Which of the following is responsible for medication regulation?
A. ✅SFDA
B. MOH
C. SCFHS

D. Pharmacy and Therapeutics Committee


Validity of diazepam prescription is ? 7 days
Patient is using Metformin + Sitagliptin +pioglitizen What is the most appropriate monitoring for him?
A) Pulmonary function test
B) Renal function test
C) ✅Liver function test ypi
furosemide infusion in heart failure pt NYHA class 4, I guess the dose was 120 not sure ?
A- ✅30 mintues
B- 40 mintues C- 10 minutes D- 5 mintues
Natural product used with alzahimer disease:
Gingko biloba
Student whant to do sreash
Googl
Upto dat
Med
Pharmacy coll doctor for now reasrch about asmtha and urine retantion.
Pharmacotharpy
Med
Gidline
Google
Since insulin is directly involved in the uptake of glucose into tissue, and that glucose uptake by
muscle can increase seven- to twentyfold during exercise insulin concentration decreases during
exercise of increasing intensity.
- Aim of krebs cycle? ATP production
- Exercise induced asthma treatment
- Thiazides durtics effect in low renal functions? Has no effect
- Case about women having constipation and dark stool due to what medication? Ferrous sulphate
- Cyclophosphamide SE? Hemorrhagic cystitis
- Lots of questions about EGC
- LD calculation
- Naloxon pediatric dose
- Treatment of uti in pregnancy
- What vaccines are contraindicated during pregnancy?
- Natural triglycerides lowering agent
- B12 name?
- Vincristine label—-> don’t administer interathecal
- Covid-19 infection control first line preventing transmission: (wearing surgical mask-n95mask-
wearing gloves-hand washing)
DNA to mRNA?
-Trancraption
-Transltion
-Trancraption
1-Plant source for treatment of malaria falciparum?
2. Aspirin structure
3. Patient needs the highest degree of care in MOH which insurance card to choice? Gold card?
Altamayoz card?
4.Quinidine structure?
5. Community pharmacist notices that there is a relation btw asthmatic patients and obesity and he
wants to make a research about this! collect information from where??
6.antidote for methotrexate
7. Long case, the pt was use 4 antiviral +floconazol and he UGT1A1 +ve which drug should draw out?
‫ ﻣﺎﻓﻲ‬Irinotecan !
8. ‫ﻗﺼﺔ طﻮﯾﻠﺔ ﻧﮭﺎﯾﺘﮭﺎ اﻧﻮ اﻟﺒﯿﺸﻨﺖ ﺟﺎ ﺻﺮخ واﻧﻔﻌﻞ ﻓﻲ ﺻﯿﺪﻟﻲ وھﻮ طﻠﺐ اﻷﻣﻦ ﻋﺸﺎن ﯾﺠﻮ ﯾﺎﺧﺪوه! اﻟﺴﺆال اﯾﺶ ﻛﺎن ﻣﻔﺮوض‬
‫اﻟﺘﺼﺮف اﻟﺼﺤﯿﺢ ﻟﻠﺼﯿﺪﻟﻲ؟‬
٩. ‫اﺳﺘﺮﻛﺸﺮ ﻏﺮﯾﺐ اﯾﺶ ﻧﺰود ﻋﺸﺎن ﻧﺰﯾﺪ اﻻﻣﺘﺼﺎص؟‬
١٠. Which one need dose increase in elderly? Levothyroxine? Chloropromazine ? ‫وﺧﯿﺎرﯾﻦ ﻛﻤﺎن‬
١١. Pt have asthma came to hospital for something else, normal lab results except high potassium!
Which one cause this? SABA? Ipratropium? budesonide?
12. Primidone pro drug for?
13. Baby with toothache, which one is Blackbox for babies to use! Lidocaine gel? Clove oil? And two
more options
14. Child with growth retardation and ‫ﻛﺴﺎح‬
and low calcium what vitamin to use?
15. Obese pt wants to use Orlistat
‫اﻋﻄﯿﮫ واﻛﻠﻤﻮ ﯾﺎﺧﺪ ﻓﯿﺘﺎﻣﯿﻨﺎت؟‬
‫ﻣﺎ اﻋﻄﯿﮫ واﻧﺼﺤﮫ ﯾﺨﻔﺾ وزﻧﮫ ﺑﻄﺮق ﺛﺎﻧﯿﮫ؟‬
‫ﻣﺎ اﻋﻄﯿﮫ واﻧﺼﺤﻮ ﯾﻌﻤﻞ ﺗﻜﻤﯿﻢ؟‬
١٦. Alcohol percent in USP? ‫ﻓﻲ اﻟﺨﯿﺎرات ﻓﻲ ﻧﻮﻋﯿﻦ اﻟﻜﺤﻮل ﻣﻊ ﻧﺴﺒﮭﻢ‬
١٧. Accepted potency in shelf life?
١٨. If you want to make a Compounding where to search?
‫ ﻓﻲ ﺧﯿﺎر‬USP
١٩. The safest way to write prescription? Omeprazole 20 mg Bid? and 3 other options ‫ﻓﻲ ﺧﯿﺎرات اﻟﺘﺮﻛﯿﺰ‬
‫ﺑﺎﻟﻤﺎﯾﻜﺮو وﺑﺎﻟﯿﻮﻧﺘﺲ‬
٢٠. Chid took a steroid for 5 days and now the vaccine time? Take or not?
٢١. Which IG appears after recent infection? No Ig M in options also no Ig G!
‫اﺧﺘﺮت ال‬D ‫ ﻻﻧﻮ ﺗﻘﺮﯾﺒﺎ ﯾﺸﺘﻐﻞ ﻣﻊ ال‬M ‫وﷲ اﻋﻠﻢ‬
٢٢. ‫ اﻋﺮﻓﻮا ﺟﺮﻋﺎت اﻷدوﯾﺔ ﻓﻲ ال‬Heart failure
‫ﻻﻧﻮ ﺑﻌﻄﯿﻜﻢ اﻷدوﯾﺔ واﻟﺒﯿﺸﻨﺖ ﻣﺎ ﺑﺘﺤﺴﻦ ﻣﻔﺮوض ﯾﺰﺑﻂ اي ﺟﺮﻋﺔ ﻓﯿﮭﻢ ؟‬
٢٣. Pt on stage 4 heart failure still not controlled and all drugs doses are OK what is next step? Add
spironolactone?
٢٤.which drug use in both ventricular and supra ventricular arrhythmia? Dofetilid?sotalol? No
amiodarone in choices
٢٥. Graph of Pka where is the highest absorption of drug
٢٦. Pt use pioglitazone should monitor what?
٢٧. Diabetics should check for micro vascular damage every?
٢٨.women HCG + should avoid?
٢٩. Minor and deep injury vaccine
٣٠. Long case and at the end pt have severe hypotension what is the initial therapy? Hypertonic saline
٣١. Long case and all drugs are OK but there is no drug prescribed for arrhythmia!
٣٢. Women with osteoporosis and heart burn what will benif for both? Ca carbonate
٣٣.‫ ﺑﯿﺸﻨﺖ ﺟﺎ ﯾﺤﻜﻲ ﻟﺼﯿﺪﻟﻲ اﻧﻮ ﺑﯿﺴﺘﺨﺪم ﻣﻊ زوﺟﺘﻮ ال‬lorazepam ‫ وﺗﻘﺮﯾﺒﺎ ادﻣﻨﻮ وﻗﺎم اﻟﺼﯿﺪﻟﻲ ﻋﻤﻞ‬documentations ‫ﻟﻠﺤﺎﺟﮫ‬
‫دي! اﯾﺶ رأﯾﻚ ﻓﻲ ﺗﺼﺮﻓﻮ؟ ﻓﻲ اﻟﺨﯿﺎرات اﻧﻮ اﻟﺼﯿﺪﻟﻲ ﻣﻔﺮوض ﯾﺤﺘﺮم ﺧﺼﻮﺻﯿﺔ اﻟﻤﺮﯾﺾ وﻣﺎ ﯾﺤﻜﻢ ﻋﻠﯿﮫ‬
٣٤. Abstract of a research, read the result and what you recommend?
‫ﺧﻼﺻﺔ اﻟﺒﺤﺚ ﻛﺎن اﺳﺘﻄﻼع ﻟﻠﺒﯿﺸﻨﺘﺲ ﻋﻦ رأﯾﮭﻢ ﻓﻲ ﻣﻮت اﻟﺮﺣﻤﺔ وﺗﻘﺮﯾﺒﺎ اﻷﻏﻠﺒﯿﮫ اﺧﺘﺎرو اﻧﻮ اﻟﻄﺒﯿﺐ ﯾﺨﺘﺎر ﻟﮭﻢ وﯾﻨﻮب ﻋﻨﮭﻢ‬.
‫ اﻟﻤﮭﻢ اﺧﺘﺮت‬more patients education.
٣٥. Pt came to hospital with severe renal failure
‫ﻣﻔﺮوض ﯾﻮﻗﻒ ﺣﺎﻻً اﯾﺶ؟‬
‫اﺧﺘﺮت ال‬metformin
٣٦. Before dye contrast stop what drug?
٣٧.‫ﻣﻮاﺻﻔﺎت ﻣﻮﻗﻊ اﻟﺼﯿﺪﻟﯿﺔ اﻟﻤﺜﺎﻟﻲ؟‬
٣٨. ‫ ﺻﺎﺣﺐ ﺳﻠﺴﻠﺔ ﺻﯿﺪﻟﯿﺎت ﻋﺎﯾﺰ ﯾﻌﯿﻦ ﺻﯿﺎدﻟﺔ ﻗﺎم ﺳﺄﻟﻚ ﻓﻲ اﻻﻧﺘﺮﻓﯿﻮ اﻧﻮ ﺗﻀﻊ ﺧﻄﺔ ﻣﻨﺎﺳﺒﺔ ﻟﻞ‬budget ‫او ﺣﺎﺟﺔ زي ﻛﺪه‬
٣٩. Which drug counteracts the respiratory depression after anathesia? ‫ ﻓﻲ ﺧﯿﺎر‬picrotoxine
٤٠. Structures with different in 2 chiral center? Isomer
٤١. Patient use heparin and aPTT 140 seconds? What to do?
٤٢. Calculation of phenytoin normal level and in lab results low albumin?
٤٣. Pt on vancomycin 1 g every 12 hrs and the trough less than 10 what to do? Increase dose? Increase
frequency of dose?
٤٤.Case pt have MI,
‫ ﯾﺎﺧﺪ‬aspirin 81 bid + ticagrelor
Or aspirin once +ticagrelor bid
‫ واﻟﺨﯿﺎرﯾﻦ اﻟﺜﺎﻧﯿﯿﻦ ﻧﻔﺲ اﻟﻔﻮق ﺑﺲ ﻣﻊ ال‬prasugrel ‫ﻣﻊ اﻟﺠﺮﻋﺎت‬
٤٥. Which one is evidence based?
‫وﺟﺎﯾﺐ اﺳﺘﺨﺪاﻣﺎت اﻷﺳﺒﺮﯾﻦ ﻓﻲ ﺣﺎﻻت ﻣﺨﺘﻠﻔﺔ‬
٤٦. Long St segment in ECG indicate what?
٤٧. ‫ أﻋﻄﺎﻧﻲ ﺗﺎرﯾﺨﯿﻦ واﺣﺪ اﻛﺴﺒﯿﺮي ﺣﻖ اﻟﺪواء واﻟﺘﺎﻧﻲ اﻟﯿﻮم اﻟﻠﻲ ﺑﺴﻮي ﻓﯿﮫ‬unit dose
‫اﺣﺴﺐ‬
A researcher gathered all studies from the past ten years. Vitamin E was used for a variety of condition.
The populations studied as well as the vitamin E formulations and doses were all different. The
researcher compared the incidences of cardiovascular-related mortality in those taking vitamin E
.supplements versus those that did not
Which of the following best describe this type of study :
A. Meta-analysis ✅
B. Cohort study
C. Observational study
D. Controlled clinical trial

-23year's old female didn't exposed to the sun two months ago, when she exposed for 2min she got her
face sun burn, doctor prescribed her sun block and when she applied it for 18 min she got the same
degree of sun burn but in her back.
What's the calculated sun block SPF?
_SPF 7
_SPF 9
_SPF 10
_SPF 15
‫ﺳﺒﻖ وﺷﻔﺖ ھﺎﻟﺴﺆال ع اﻟﻘﺮوب وﻛﺘﺒﺘﮫ ﺑﺎﻟﻨﻮت ﻻن ﻓﻌﻼ ﺟﺎﻧﻲ وﻻاﻋﺮف اﺟﺎﺑﺘﮫ ﻟﻸﻣﺎﻧﺔ‬

-antidote for (paracetamol, organophosphate insecticide, sevelamer for hyperphosphatemia, )


-structures(1-which structure should be avoided in allergies penicillin patient,2-which one of structure
is non dihydropyridine of CCB , 3- structure about Epimer )
‫ ﺣﺎﻟﯿﺎ اﺗﺬﻛﺮ ﺑﺲ ھﺬي وﻓﻲ اﺳﺌﻠﺔ ﺗﺠﯿﺐ ﺳﺘﺮﻛﺘﺸﺮ واﺳﻢ اﻟﺪواء ﺛﻢ ﯾﺴﺎل ﻋﻠﯿﮫ اﻣﺎ ﻓﻲ‬S/E ‫او اي ﻣﻌﻠﻮﻣﺔ ﯾﺨﺺ اﻟﺪواء‬

-unit of cost utility :


QALY✅
-scenario of cost effectiveness ✅ , scenario of cost binifit , which one is use in natural units
-defined planning , organization
-which one is as quantitive research :
A)analytical research ✅
B)describtive research
C)clinical trial
D)RCT
-which one of Antibiotics is high
sensitivity (by plate of microorganisms)

-which one of drugs cause increase of warfarin effect or INR effect

-which one of disease cause


Atherosclerosis (I choose
hypothyroidism) ،duration of monitor levothyroxin

-dose of iron in anemia:


A)100 mg TID/daily ✅
B)325 mg TID/daily
‫اﺣﺘﺮت ﺑﯿﻨﮭﻢ ﺻﺮاﺣﺔ‬

-S/E of ciprofloxacin in child (other anemia):A)osteoporosis, B)Athropathy


C) tendonitis ✅
‫اﻟﺒﺎﻗﻲ ﻧﺴﯿﺘﮫ وﻣﺎﻋﺮﻓﺖ ﺻﺮاﺣﮫ ﺑﺲ اﺧﺘﺮت اﻟﺜﺎﻧﯿﺔ وﺗﺎﻛﺪوا‬

-scenario about Autonomy(‫) ﻣﺎﻛﺎﻧﺖ ﺻﺮﯾﺤﺔ ﺣﺘﻰ ﺑﺎﻻﺟﺎﺑﮫ‬and justices

-disease :
‫ ﻛﺎن ﺷﺎﻣﻞ ﺻﺮاﺣﺔ ﻣﺎﻛﺎن ﻣﺤﺪد ع ﺷﻲء ﻣﻌﯿﻦ ﻛﺎن ﻓﻲ ﻛﺎﻧﺴﺮ و‬ACS ‫ و اﻋﺮاض‬COPD ‫و‬UTI ، ‫ﺑﺲ رﻛﺰوا اذا اﻟﻜﯿﺲ ﺣﺎﻣﻞ او ﻻ‬
‫ وﺟﺎب ﻛﯿﺲ ﺗﺒﻊ‬hepatic cirrhosis ‫ﺑﻤﺎ ﻛﺎﻧﺖ ف ﯾﻮﺣﺪه ﺗﺸﺮﺣﮭﺎ ﺑﺎﻟﺜﺮوب اذا اﻧﺰﯾﻤﺎت اﻟﻜﺒﺪ ﻣﺮﺗﻔﻌﮫ ﺑﺤﺎﻟﮭﺎ اﯾﺶ ﯾﻌﻨﻲ واذا ﻣﺮﺗﻘﻌﮫ‬
‫ ﻣﻌﺎھﺎ‬bilirubin ‫اﯾﺶ ﺗﻌﻨﻲ‬

- when use medications in ACs , which one of drug need to monitor ?aspirin , clopidogrel , statin ✅,
lisinopril , BB
‫ ھﻲ ﻧﻔﺲ اﻻدوﯾﺔ ﺗﺒﻊ‬STEMI -angina
- case about disease:
‫اﻟﺨﯿﺎرات ﻛﺎﻧﺖ اﻧﻮاع اﻟﺬﺑﺤﺔ اﻟﺼﺪرﯾﺔ اي وﺣﺪه ﻣﻨﮭﻢ ﻛﺎن اﻟﻤﺮﯾﺾ ﯾﻌﺎﻧﻲ ﻣﻨﮭﺎ‬

-temperature (mesophiles for M.O , Refeagitator room , Room temperature, biological product is
storage in what ? (2-8) , which one is considered as bacteria ? D

-vaccine(which the one is Live vaccine ? , other name of HPV vaccin (Gradasil ) , when the Baby
delivered with mom has Hap B what to give him ?, which type of cell produce lymphocytes? B-
lymphocyte

-which one as ondanerserton MOA :


I choose ginger
-calculation : different infusion rate , one of correction phenytoin level
‫ﺻﺮاﺣﺔ ﺑﻌﻀﮭﺎ وﷲ ﻣﺎﻋﺮﻓﺖ اﺣﻠﮭﺎ ﻣﺎھﻲ واﺿﺤﺔ وﻻﻣﺒﺎﺷﺮة ﻟﻜﻦ ان ﺷﺎءﷲ ﺧﯿﺮ‬

-medication used in Respiratory ..? Pavalisuzumab

-dose of methotrexate and how to take with folic acid

-abbreviation of O.U gtt. BID (about drops Eyes , Ears ) ‫ﯾﺠﯿﺐ ﻣﺜﻞ ﻛﺬا ﻣﺜﺎل وﺻﻔﺔ وﯾﻘﻮل وش ﻣﻌﻨﺎھﺎ ﺑﺎﻟﺠﻤﻠﺔ‬

-primary function about golgi in human :


A)produce ATp
B) transportion ..

phases arrhythmia (P wave , QRS complex )

-different scenario about practice and regulations, I cannot remember them

-which one of drug contraindicated to pregnant : atenolo ✅, amlodipin


‫اﻟﺒﺎﻗﯿﻦ ﻣﺎاذﻛﺮھﻢ‬

-MOA dantrolen
I
-norepinephrine (hormone , neurotransmitter)

-‫ﻓﻲ ﺻﻮرة ع اﻟﻘﺮوب ﺗﺒﻊ اﻻﺳﮭﻢ ﻻدوﯾﺔ اﻟﻀﻐﻂ‬

٪١٠ ‫ ﻟﻜﻦ ﻻﺗﺸﯿﻠﻮن ھﻤﮫ ﺑﻤﺎ ان ﻧﺴﺒﺘﮫ‬، ‫ﻟﻼﻣﺎﻧﮫ‬-‫ﻓﻲ ﻓﺴﯿﻮﻟﻮﺟﻲ وﻣﯿﻜﺎﻧﯿﺴﻢ ﻟﻜﻦ ﻣﻮ ﻣﺘﺬﻛﺮﺗﮭﻢ‬
‫ ﺳﯿﻨﺎرﯾﻮا ﻋﻦ‬.‫ رﻛﺰوا ع اﻻھﻢ ﻻن اﻏﻠﺒﮫ‬social /administrator ‫واﻟﺜﯿﺮاﺑﻲ واﻟﻤﺴﺎﺋﻞ ﻗﻠﯿﻞ اﻟﻠﻲ داﯾﺮﻛﺖ اﻏﻠﺒﮭﺎ ﯾﺒﻲ ﻟﮭﺎ ﺗﺮﻛﯿﺰ‬

-‫ وﻓﻲ ﺳﺆال ﻋﻦ‬enzume inducer and inhibitors by cimitidine ‫اﯾﺶ ﺗﺎﺛﯿﺮ ع اﻟﺪواء اﻟﻔﻼﻧﻲ وھﻜﺬا‬

-which drug cause dyspepsia (dabigatran ) ✅


-‫ ﺟﺎء ﺳﺆال واﺣﺪ ﻋﻦ‬SFDA ‫ﻟﻜﻦ ﻣﺎاذﻛﺮه‬

-Disadvantages of primary literature

-defined drug advers events

-which type of vit D is good for renal failure patient ?


Calcitirol

-question about expiration date of approval drug product after 1996( i think after 20years , i choosed
2016 , but there are othe option is 2024) i am not sure

Elongation and termination in cells


‫ﺟﺎء ﻋﻠﯿﮭﻢ ﺳﺆال ﻓﻲ اﻟﺨﻠﯿﮫ وﻗﺎل اﯾﺶ اﺳﻢ اﻟﻌﻤﻠﯿﺔ وﻛﺎﻧﺖ ﺑﯿﻦ اﻟﺜﻨﺘﯿﻦ ھﺬي ﻻﺑﺪ ﻣﻦ ﻣﻌﺮﻓﺔ ﻣﻌﻨﺎھﻢ‬

‫ اﺗﻮﻗﻊ اﻧﮭﺎ ﻛﺎﻧﺖ‬penetration of cell wall


‫ واﻟﻌﻤﻠﯿﺔ‬Elongation ‫ﺑﺲ ﺗﺎﻛﺪوا‬

Type alcohol in USP pharmacopoeia


Metabolite of diclofenac by structure

Question about INR, A person with INR of 12 and given 10 mg of vitamin K when will you
measure INR?

Encapsulated bacteria?
E.coli
Narcotic addiction treatment?
Methadone
What is the medication that is safe in pregnant women with eplipsy?
Lamotrigine
Drug treatment of Pylori?
PPP + Amoxicillin + Clarithromycin
Clopidogrel stop before surgery?
5 days
Vaccine contraindicated in pregnancy?
MMR
Live attenuated vaccine?
Rubella
Side effect of isosorbide dinitrate?
Orthostatic hypotension
Drug that is beta blocker and partial beta agonist?
Pindolol
Content of stone in gall bladder?
Cholesterol
Definition of planning strategy?
Patient in her 20s has not taken a vaccine for 11 years, what vaccine would you give her?
Tdap
Meningitis
Flu
Structure of acyclovir?

Choose a drug ( structure) for patient with sulfa allergy?

Azole structure?

Patient with constipation, lab values are normal except for Hct and has dark stool. What is the
cause vitamin B12 or ferrous sulfate?

Structure of SNRI, what to add on SNRI to make it more selective?

Case… Hyperthyroidism:
Elevated TSH, increased T4
Increased T4, elevated TSH
Elevated TSH, decreased T4
Elevated T4, decreased TSH
Phase 2 metabolism conjugation?

Structure, which racemic mixture contain chiral center?

Oxacillin and 4 structures, what is added to the structure that make it b-lactamase resistant?

Structure, which has anti pseudomonal activity?

Structure antivirals, marked 4 area on structure and asked about which part to remove that make
the antiviral drug not effective?
N3
CH3
C=O
Which has double bond?
CO2
C2H4
Medication require amber glass?

Case patient with GERD what is patient education?


Contact physician if you have flu like symptoms
Stop if you have hypoglycemia
Follow up regularly to check electrolyte
Take it for 1 month
Diabetics patient with heartburn, what to give him?

Patient with Isoniazid patient education?

Ondansetron acts as anti-emetic, which from these substances can exhibit same effect?
ginger
clove
cinnamon
black liquorice
use for dyslipidemia?
Fish oil

best ttt of antacid for patient with osteoporosis ?

Calcium carbonate
‫ ﺳﺘﺮﻛﺸﺮز وﺳﺄل وش‬٤ ‫ ﺟﺎب‬H1 blocker ? ‫ﺑﺲ ﻓﻲ اﻟﺨﯿﺎرات ﻛﺎن ﻛﺎﺗﺐ اﺳﻢ اﻷدوﯾﺔ‬

‫ و اﻟﺠﻮاب ﻛﺎن‬..‫ ﻣﺮﻛﺒﯿﻦ‬epimers

‫ ﺟﺎب ﻛﯿﺲ و ﺳﺄل وش ﻧﻮع اﻻﻧﯿﻤﯿﺎ ﻋﻨﺪ اﻟﻤﺮﯾﺾ؟ ﻛﺎن ﺣﺎط اﻟﻼب ﻓﯿﮭﺎ‬..

Case patient with pneumonia. and +ve H1N1..what to give ?


- Acyclovir
- Oseltamivir
What drug that increases prolactin?

flaccid paralysis?

patient with AF and MI.. what anti arrhythmic medication that is contraindicated?
- sotalol
- Amiodarone
- Domperidone
Case hyperlipidemia .. ‫ و اﻟﺸﻲ اﻟﺰاﯾﺪ ﻋﻨﺪه ھﻮ‬..‫ و ﻛﺎن ﺣﺎط اﻟﻼب ﺣﻖ اﻟﻤﺮﯾﺾ‬TG .. ‫و اﻟﺴﺆال وش ﻧﺴﺘﺨﺪم ﻟﻠﻤﺮﯾﺾ؟‬
‫ ﻛﺎن ﻣﻦ اﻟﺨﯿﺎرات‬cholestyramine

If we give the patient live attenuated vaccine.. the the patient will develop which immunity?
Natural active immunity
Artificial active immunity
Passive immunity
patient is taking orlistat to decrease her weight. What to advice her ?
Take multivitamins
Take with isoniazid ?
Pyridoxine
decrease CVD in DM ?
‫ ﻛﺎن اﻟﺠﻮاب دواء ﻣﻦ ﻣﺠﻤﻮﻋﺔ‬SGLT-2 inhibitor
‫ دراﺳﺔ ﻋﻦ‬cacagliflozin .. 12.4 ‫ و اﻟﺒﻼﺳﯿﺒﻮ‬11.7 ‫و ﻛﺎن ﻣﻦ ﺿﻤﻦ اﻻرﻗﺎم ان اﻟﺪواء ھﺬا ﻧﺴﺒﺔ اﺻﺎﺑﺘﮫ ﺑﺄﻣﺮاض اﻟﻘﻠﺐ‬
‫و ﺣﻂ ﺧﯿﺎرات‬
- canagliflozin cause CVD
- canagliflozin is protective against CVD .
‫ دراﺳﺔ ﻋﻦ ﻣﺮﺿﻰ ﺳﺮطﺎن ﻓﻲ ال‬Stage ‫اﻻﺧﯿﺮ‬..
‫ و ﻛﺎن ﯾﺴﺄل ﻋﻦ ال‬conclusion ‫وش ھﻲ؟‬
Case .. DM , A1c 7.. on metformin 825 BID and not controlled .. what to do next?
Aspirin is not safe to give to children :
Less than 2
Less than 6
safest route for administration for infants ?
‫ و ﻛﺎﻧﺖ اﻻﺧﺘﯿﺎرات ان ﻧﻌﻄﯿﮫ اﻟﺪواء ﺑﺎﻟﻤﻠﻌﻘﺔ او اﻟﻜﻮب او‬syringe
1-month year is :
- neonate
- infant
4-year-old girl with sore throat. She denies fever, or any other symptoms. What to give her ?
- gargle with salt and warm water
- Amoxil.
-Augmentin
- Ciprofloxacin
‫ ﻛﯿﺲ ﻟﻤﺮﯾﺾ ﯾﺎﺧﺬ‬methadone .. ‫ و اﺧﺬ ﺑﻌﺪھﺎ ادوﯾﺔ‬X ‫ و‬Y ‫ و‬Z ..
‫ و ﻋﻄﺎﻧﻲ ﻓﻲ اﻟﻜﯿﺲ وش ﺳﻮا ﻛﻞ دواء‬..
‫ اﻻﺧﺘﯿﺎرات ﻛﺎﻧﺖ ﻛﻠﮭﺎ ان‬X ‫ ھﻮ ﻣﻮرﻓﯿﻦ‬..
* ‫ ال‬Y ‫ ﻛﺎن اﻣﺎ‬naloxone ‫ او‬naltrexone .. ‫ وﻛﺎن ﻛﺎﺗﺐ ﻋﻦ ال‬withdrawal
*‫اﻣﺎ ال‬Z ..‫ ﻣﺮﻛﺐ ﺛﺎﻟﺚ ﻛﺎﻧﺖ اﻟﺨﯿﺎرت‬pentazocin ‫او دواء ﺛﺎﻧﻲ‬
5 alpha reductase inhibitor is ?
Finasteride
‫ ﻛﺎن اﻟﺴﺆال ﻋﻦ اﺛﻜﺲ و اﻟﻤﻔﺮوض اﻟﺠﻮاب‬beneficence.
‫ﻟﻜﻦ ﻓﻲ اﻟﺨﯿﺎرات ﻛﺎﺗﺐ‬
- Ethics and beneficence
- beneficence and nonmaleficence
Best BDZ for patient >75?
Alprazolam
which is best for risk reduction ?
- Rules and policies
- Forcing functions
lipid soluble vitamins? KEDA
loperamide is? opioid agonist on intestine
diagnosis of STEMI ?

- elevated ST segment with increased troponin ✔


- elevated ST segment with increased bnp
- elevated ST segment with increased CK
‫ ﺟﺎ ﺳﺆال ﻋﻦ‬BPH ‫وش ﻧﻌﻄﻲ اﻟﻤﺮﯾﺾ‬
GFR 50 , the patient is in which stage ?
Stage 3
cytotoxic hypersensitivity? Anemia
Calculate therapeutic index
‫ اﻧﻚ ﺗﺸﺘﻐﻞ ﻓﻲ‬، ‫ ﺳﯿﻨﺎرﯾﻮ‬community pharmacy ..‫ وش ﺗﺮاﻋﻲ ؟‬، ‫و ﺟﺎي ﺑﺘﺸﺘﺮي ﻣﻨﺘﺞ ﺟﺪﯾﺪ‬
- brand and generic
- the needs for the people that the pharmacy serve.
‫ ﺟﺎب ﻛﯿﺲ و ﻧﻔﮭﻢ ﻣﻨﮭﺎ ان اﻟﻤﺮﯾﺾ ﻋﻨﺪه‬septic shock..
Which fluid we should give ?
- ringer's lactate ✔
- hypertonic saline
Poor oral bioavailability? Vancomycin
decreased bioavailability:
decrease absorption
decrease distribution to brain
Patient is taking vancomycin and his level is 23 (normal 10-20) ,
‫و ﻗﺎل ان اﻟﻤﺮﯾﺾ ﺑﺎل‬level ‫ و اﻟﺴﺆال وش ﻧﺴﻮي ؟‬.. ‫ذا ﻋﻨﺪه اﻓﻜﺖ‬
- decrease the vanco dose
- increase Vanco dose
- do not change the dose
- change the drug
‫ ﻣﺜﻼ ﯾﻘﻮل ان اﻟﻤﺮﯾﺾ ﯾﺎﺧﺬ‬.. ‫ ﻓﻜﺮة اﻻﺳﺌﻠﺔ‬vanco ..‫ف وش ﻧﺴﻮي ؟‬.. ‫و ان اﻟﻤﺮﯾﺾ ﻟﺴﻰ ﻣﻮ ﻛﻨﺘﺮوﻟﺪ‬
-‫ﻧﺰود اﻟﻔﺮﯾﻜﻮﻧﺴﻲ ﻟﻠﻔﺎﻧﻜﻮ‬
-‫ او ﻧﺨﻠﯿﮫ‬extended release
- ‫او ﻧﺰود اﻟﺪوز‬
‫ اﺧﺘﺮت ﻧﺰود اﻟﻔﺮﯾﻜﻮﻧﺴﻲ ﻻﻧﮫ‬time-dependent

Question about toxicity, patient came with non-productive cough , dyspnea, fever:
Which one of the following is most likely to be the cause ?
Amiodarone
Uniformity test applied to determine the content of the ingredient in tablets:
-Bioequivalence
-Dissolution
-Purity
-Potency
UGT1A1 allele;
- Fluconazole
- ...Navir
- ‫ اﻟﺸﻲ اﻟﻠﻲ ﯾﺨﻠﻲ اﻟﻤﺮﯾﺾ ﯾﺴﮭﺐ و ﯾﺘﻜﻠﻢ ؟‬/
-Close ended questions
-face expressions
-body posture
Contraindicated together?
Aliskiren and lisinopril
The test coagulase is to differentiate between?
S. aureus and S.epidermidis
‫ ﺳﺄل ﻋﻦ اﻟﺒﻜﺘﯿﺮﯾﺎ اﻟﻠﻲ ﺗﺴﻮي‬hemolysis .. ‫ ﺑﺲ ﻣﺎ اﺗﺬﻛﺮ اذا ﻛﺎن‬alpha or beta

‫ﺳﺆال ﻋﺎم ﻋﻦ ال‬


staining to gram positive bacteria " staph aureus" :
‫واﻟﺨﯿﺎرات ﻛﺎﻧﺖ ان‬
- purple because the thick layer retains the crystal violet stain
- purple because the capsule
Pneumonitis is side effect of ?
Bleomycin
which antibiotic cause hyperkalemia
- ciprofloxacin
- Norfloxacin
- Trimethoprim/sulfamethoxazole
- Amoxicillin
‫ اﯾﺶ اﻟﺪواء ﻣﻦ ادوﯾﮫ‬osteoporosis ‫ﯾﻨﺄﺧﺬ ﻣﺮه وﺣﺪه ف اﻟﺸﮭﺮ‬
Calculate prevalence?
‫ ﺟﺎﻧﻲ ﺳﺆال‬DM type 1 ‫ او‬%. %75‫ ﯾﺼﯿﺮ ﺑﻌﺪ‬after complete destruction ‫ﺑﺼﺮاﺣﮫ اﺣﺘﺮت ﻓﯿﮭﺎ‬
Anticoagulant in cancer patient
‫ اﻟﻌﺎﻣﻞ اﻟﻠﻲ ﯾﺄﺛﺮ ﻋﻠﻰ اﻣﺘﺼﺎص‬vitb12 ‫ اﻟﻠﻲ ھﻮ‬intrinsic factor
‫ﺟﺎب ﻛﯿﺲ اﻧﻮ اﻟﻤﺮﯾﺾ طﻠﻊ ﻋﻠﯿﮫ اﻋﺮاض ﻏﺮﯾﺒﮫ ﻣﻦ دواء ﻣﻌﯿﻦ ﻣﻊ اﻧﮫ ﻣﺎﯾﺎﺧﺬ اي دواء ﻣﻌﺎه وﯾﺒﻲ ﯾﺒﺤﺚ ﻋﻦ اﻟﺴﺒﺐ وﯾﻦ ﯾﺒﺤﺚ‬
‫ ﺣﻄﯿﺖ‬myler
‫ ﺟﺎء ﻛﯿﺲ اﻧﻮ اﻟﻤﺮﯾﺾ ﺻﺎر ﻟﮫ‬red man syndrome
1000 ‫اﻟﻔﺎﻧﻜﻮ ﻛﺎن‬mg in 30min
‫اﯾﺶ اﻟﻤﺎﻧﻨﺠﻤﻨﺖ ھﻨﺎ‬
Dec dose
Dec infusion time
Inc infusion
Inc the dose
‫ ﺟﺎﻧﻲ ﻣﺎﯾﻜﺮواورﻗﺎﻧﺰم‬anaerobic obligations
Statin taken in the night? Simvastatin
Amino acid in Rheumatoid arthritis ‫ﻣﺎﻧﻲ ﻣﺘﺄﻛﺪه ﻣﻨﮫ‬
Obese pt with DVT what is the treatment?
pregnant pt with DVT? Enoxaparin
Calculate allegation.
Natural products work as anticough?
Calculate Iv bolus?
‫ اﯾﺶ ﻣﻦ أدوﯾﺔ‬DM ‫ﯾﺴﻮي‬
Hepatoxicity?
Pioglitazone
Metformin
DM 1 patient, give what?
Sulfonylurea
GLP1
DDP
Pioglitazone
No insulin option

which phase has minium people


2- phase that conduct the unexpected adver drug reaction
3-infection phase prodrome defintion
4- om salm sale herbal medcin for dm and pepole complaine
5- Theraputic index aim to ?
6-variation between drug freqancy and potency
7-emulsion and suspsion defintion
8- morbity defintion
9- ‫ ﺟﺎء ﻛﺜﯿﺮ ﻣﻦ‬cost ‫ اﺳﺌﻠﺔ‬10 ‫ﺗﻘﺮﯾﺒﺎ‬
10 - toxidrom defintion
11- minor injury with unknow history
12 deep injury with unknow history
13 - patinet eats penute and have swelling face deficult to breath which type of reaction ?
Hypersensvtie
14- in hypersnsvite which appers ? Histamin
15-non esstional amino acid
16 - vit k adminstration route for infant
17 - look alike soud alike solution ? Tall man letter
18 -‫ ﺟﺎء ﻛﺜﯿﺮ ﻣﻦ‬Ig ‫ ﻣﺜﻞ‬which one cross the placenat
19 - have gamma chain ? Ige
20- which one appers in hypersensvti ? Ige
21 - codien to morphen sar
22- ‫ ﺟﺎب وش ﻧﻐﯿﺮ ﻓﻲ‬SNRI ‫ ﻋﺸﺎن ﯾﺼﯿﺮ‬selective Serotene inhbitor
23- asprin sar
24- T - lymphod ‫ﺗﺘﺼﻨﻊ وﯾﻦ‬
25 - what hormone secreted from antior pituitory gland ‫ﺟﺎء ﺳﻮاﻟﯿﻦ ﻧﻔﺲ اﻟﻄﺮﯾﻘﺔ‬
26-clostirduim ? Gram + Rod anarobic or arobic
27- ‫ ﺗﻌﺮﯾﻒ‬catacory c for preganat
28- p value ? Answers was > siganficant clincal or statically
‫ﯾﻌ ﻨ ﻲ ا ذ ا ﻛ ﺎ ﻧ ﺖ ﻛ ﺒ ﯿ ﺮ ه ﺗ ﻌ ﺘ ﺒ ﺮ ﺳ ﻘ ﻨ ﻔ ﻜ ﺎ ﻧ ﺖ ا و ا ذ ﻛ ﺎ ﻧ ﺖ ﺻ ﻐ ﯿ ﺮ ه ﺗ ﻌ ﺘ ﺒ ﺮ ﺳ ﻘ ﻔ ﻨ ﻜ ﺎ ﻧ ﺖ‬
‫ وﺧﯿﺮات ھﻞ ﺗﻌﺘﺒﻰ‬clincal or statical

29-‫ ﺟﺎء ﻋﻦ‬scfhs ‫ھﻲ اﻟﻠﻲ ﺗﻌﻄﻲ اﻟﺮﺧﺺ ﻟﻠﻤﮭﻨﮫ‬


30 - drug inhibtor prolactin
31- woman with hypertension what is proper oral contrrceptive for her
32- carbidopa function in levadopa drug ?
33- which drug when we give we montering the heart 6 houre later ?‫ اﺧﺘﺮت‬fingolimed
34- NISAD ACTION ON STOMACH ?
35- captopril SE ? Defect in test
36- captopril SE ? DRY COUGH
37- Direct inhibtor of thrombin ?
38 -chlamydia treatment ? Azithromycin
39- which one can displace the spironlacton ? K or Na or mg
40 -warfaren was add ob levoflocxine for Respirtory infection ? Drug -drug interaction or drug -
food interaction
41- metoformine SE ? lactic
42- to treat patient with tb what is the bast way ? DOT
DIRCTe observation treatment
45 -which can increase risk of heart dises when take alone ? Short actin B agonst or long acting
antichloinrgic or corticosroid
46- heparin antidote
47-benzodiazpen antidote
48- which antibiotic cause c.deffic ? ‫ اﺧﺘﺮت‬clindmycin
49- ethamutaol TB Drug SE ‫ﺟﺎء ﻣﺮﺗﯿﻦ ﻣﺮا ﺑﺴﺆال ﻟﺤﺎﻟﮫ وﻣﺮا‬
‫ﺑﻜﯿﺲ‬
50- dispens of diltazam for ? ‫ اﺧﺘﺮت‬one month
51- isotonic solution ? Ringer
52- anemia low MCA and low hemoglobin which type ? Iron or B12 anamia
53- which one of the TB drug interaction with most of the drugs ?
54-warfarin test ? INR
55- ‫ ﺟﺎء ﻛﯿﺲ ﻣﺮﯾﺾ ﯾﺎﺧﺬ‬codien ‫ ﺟﺎء ﻟﻠﻄﻮارى ﯾﺸﺘﻜﻲ ﻣﻦ‬tchycardia palpation what the causa ‫؟‬
Rapid motbloizer of c2d6 ‫اﺧﺘﺮت‬
56- drug interact with st john wort ? ‫ ﻣﻦ اﻻﺧﺘﯿﺎرات دواء‬setralin
57- case patient 3 y.o with fever and cough with no tonsilit enalargment the docter said it could
be viral infection what give ?
a- paractamol
b- osaltamivir
58- pregnant with HIV what treament ?
59- trament of malignant hypertenstion ?
60- ‫ ﺟﺎب ﻛﯿﺲ ﻣﺮاه ﻣﻌﺎھﺎ اﻛﺘﺌﺎب وﺳﻤﻨﺖ وش اﻟﺴﺒﺐ او اﻟﺪواء اﻟﻤﺴﺒﺐ ؟‬bupropion ‫اﺧﺘﺮﺗﮫ‬
61 - ‫ ﺟﺘﻨﻲ اﻟﺼﻮره ﺣﻘﺖ‬ACE ‫ و‬ARB ‫ و‬Rennin
62- antiplatlat cauase dypensia
63 - ‫ ﻣﺮﯾﺾ ﺟﺎء ﻟﻠﺼﯿﺪﻟﯿﮫ ﻣﺎﻛﺎن ﯾﻘﺪر ﯾﺘﻜﻠﻢ ﻣﻦ اﻟﻜﺤﮫ وﻛﺎن ﯾﺎﺧﺬ دواء ﺿﻐﻂ‬linsopril ‫ ؟ اﺧﺘﺮت ﻧﺴﻮي‬refill to doctor
bec maybe the dry cough from the ACEI
64- WHICH one is correct about vit D ?
‫ اﺧﺘﺮت‬D2 and D3 inactive
65- iso 7 which room ? ‫ اﺗﻮﻗﻊ‬buffer room
66- treatment vomtiong in preganat ? B6
67-‫ ﺟﺎء ﻛﯿﺲ ان ﻓﯿﮫ ﻣﺮﯾﺾ ﻣﻌﮫ‬candiadas ‫وش اﻟﺪواء‬
a- some antibiotic treat fungl infection
b- give iv antifungl
‫ﻧﺎﺳﯿﮫ اﻟﺒﺎﻗﻲ‬
68 - what the vit c do ?
A- Decress the severtiy of the disese
B- decrese the duratoin of disese
69- ‫ ﻣﺮﯾﺾ ﺗﺠﯿﮫ اﻻﻧﻔﻠﻮﻧﺰا داﯾﻢ وش ﻧﻌﻄﯿﮫ ﻋﺸﺎن ﻧﻤﻨﻌﮭﺎ او وش ﯾﺎﺧﺬ ﻋﺸﺎن ﯾﻤﻨﻌﮭﺎ ؟ اﺧﺘﺮت‬influanza vaccine
anually ‫ﺳﻨﻮﯾﺎ‬
70 - ‫ ﻣﺮﯾﻀﮫ ﺗﺎﺧﺬ‬methotrxate ‫ ؟ اﻻﺟﺎﺑﺎت ﻛﺎﻧﺖ‬give the patient immunaztion after 2 weeks of
methotraxte ‫ او ﻧﻌﻄﯿﮭﺎ اﻣﯿﻮﻧﺰﺷﻦ ﻗﺒﻞ ﻻﺗﺎﺧﺬ‬methotraxt
71 - ‫ ﻣﺮﯾﻀﮫ ﻋﻨﺪھﺎ‬HIV ‫( وﺗﺒﻲ ﺗﺎﺧﺬ ﺗﻄﻌﯿﻢ‬vaccine )
‫ اﺧﺘﺮت‬no vaccine for her becauss she have crtical illnss
72- Rate control treatment ?
73-what is the drugs we do not give old people ?
A - amlodpine
B- metoprolol
C- metocpramid
74- which is corect about B- lactamses ? ‫ اﺧﺘﺮت‬most dose dpentent mic
75- ‫ ﻣﺮﯾﻀﮫ ﻋﻨﺪھﺎ‬ostoprosis and hypotherod ‫وﺟﺎﯾﮫ ﺗﺎﺧﺬ ادوﯾﮫ ﺳﻜﺮ وﻻﺣﻆ اﻟﺼﯿﺪﻟﻲ ھﺬا واﻗﺮت اﻧﮭﺎ ﻣﺎﺗﺎﺧﺬ وﻻ‬
‫دواء واﻧﮭﺎ ﻣﻠﺘﺰﻣﮫ ﺑﺎدوﯾﺘﮭﺎ‬
A- refill to doctor untreat patinet
B- NON adherens
C - do nothing
76- ‫ ام ﺣﺎﻣﻞ وﻣﻌﮭﺎ‬Hiptits B ‫وﻟﺪت وش ﻧﻌﻄﻲ اﻟﻤﻮﻟﻮد ﺗﻄﻌﯿﻢ ؟‬
77- what is the least important for breast feeding woman ?
A- prematuer infant
B- wight of the baby
C - drug freqancy
D- drug theraputic rang
‫اﺣﺘﺮت ﻓﯿﮭﺎ ﻣﺪري وش اﻟﺠﻮاب‬
78 - xalant ‫اﺗﻮﻗﻊ ﻛﺬا اﺳﻢ اﻟﺪواء وﻛﺎﺗﺐ وش ﯾﻌﺎﻟﺞ ؟‬
79 - high intnstiy stant
ROS 20
80- statn need montering
81- samvastatin best time take it ? At bedtime
82- ‫ وﺑﺮﺿﻮا ﺟﺎﺗﻨﻲ ﻛﯿﺲ وﻓﯿﮭﺎ ﺗﺤﺎﻟﯿﻞ اﻻ ﺗﺤﻠﯿﻞ‬phenytoin ‫ﻣﻮ طﺎﻟﻊ‬
‫ وﻣﻌﻄﯿﻨﻲ ﺑﺎﻟﺘﺤﺎﻟﯿﻞ‬albumin ‫ ﻧﺎزل ﯾﻌﻨﻲ وش اﺳﺘﻨﺘﺞ ؟‬20 ‫ﻗﻠﯿﻞ‬
A- phenytoin increase in plasma becuses decrese albumin
B- phenytoin decres becuses the decress of albumin
C- ‫ﯾﻘﻮل ان اﻟﺪواء ﻣﺎﻟﮫ دﺧﻞ ﺑﺎﻟﺒﻮﻣﯿﻦ‬
Which is the drug can adminstration intrathecal ?
Vincrstine
Vinblastin
‫ ﺑﺮﺿﻮا ﺟﺎﺗﻨﻲ ﻛﯿﺲ وﻓﯿﮭﺎ ﺗﺤﺎﻟﯿﻞ اﻻ ﺗﺤﻠﯿﻞ‬phenytoin ‫ﻣﻮ طﺎﻟﻊ‬
‫ وﻣﻌﻄﯿﻨﻲ ﺑﺎﻟﺘﺤﺎﻟﯿﻞ‬albumin ‫ ﻧﺎزل ﯾﻌﻨﻲ وش اﺳﺘﻨﺘﺞ ؟‬20 ‫ﻗﻠﯿﻞ‬
A- phenytoin increase in plasma becuses decrese albumin
B- phenytoin decres becuses the decress of albumin
C- ‫ﯾﻘﻮل ان اﻟﺪواء ﻣﺎﻟﮫ دﺧﻞ ﺑﺎﻟﺒﻮﻣﯿﻦ‬
‫ اﻟﺨﯿﺎرات ﻛﺎﻧﺖ ﻛﻠﮭﺎ‬reduction

A-reduction analgesic
B- reduction antitussive
‫ واﺛﻨﯿﻦ ﻧﺎﺳﯿﮭﻢ ﺑﺲ ﺑﻌﺪ‬reduction
13/ which drug if given intrathecal fetal ?
14/ case pt taking ferrous sulphate and black stool
15/ calculation 5 questions one of them flow rate ???
16/ what does cover the bitter taste of tablet
17 / what does make the tablet disintegrate
18/pt have toxicity and given pyridoxine ? INH
19 / definition of incidence ?
20/ pt came screaming pharmacist called the police ?
21/ Picture with ACE and ARB and determine the action
22/ drug used for HTN induce dieresis
23/ pt came and said he is using his wife diazepam
Pharmacists documented this as the pt abusing
Whats wrong ??
24/ clinical trial phase with less number of individuals? Phase 1
25/ elimination rate constant
26/ phase of clinical trial after aproval ? 4
27/ pharmacoeconimcs study use monitory units ? CBA
28/ Ethics and health regulation
29/ Almost 15-20 questions calculation in the first and second sections
30/Question about anti arrhythmic used in heart failure
31/ Calculate CHAD VAS score
32/Question about anti arrhythmias rate control
33/Live vaccine
34/ Pregnant lady with hep B what to give the child on delivery
35/ Isomer structure
36/ Creatinine clearance for female (‫)ﻧﻔﺲ اﻟﻠﻲ ﺑﺎﻟﺰﺗﻮﻧﺔ‬
37/ Patient with face swelling , side effect of which medication ? ACE
38/ When to adjust drugs renaly elemenated
Choices:
120
100
70
30
39/ patient with heart failure and DM what to avoid ? Pioglitazone
40/ elderly what to give from diazepams ?
Alprazolam
41/ PR interval represents
42/ QRST represents
43/ pre diabetic whats the intervention ? LifeStyle modification
44/ dose of calcium in pregnancy?
45 / patient with elplipcy want to get pregnant she is on vampiric acid, what to do? Change it a slowly
to levetiracetam

‫ ﺟﺎ ﺣﺴﺎب‬trough of vancomycin
Another question when you draw vancomycin ? After the 3rd dose
What happen for Clopidogrel if CYP2C19 not available
COPD category for FEV %
Calcularion incidence

‫وﺻﻔﺔ ﯾﺴﺎل ﻋﻨﮭﺎ اﻟﻤﺮﯾﺾ وﯾﻘﻮل اﺷﺮﺣﮭﺎ ﻟﻲ‬

‫ال‬high volume distribution Calculation


Medazolam antidote
Which drug cause brown in urin

‫ﺑﺲ ﯾﻌﻨﻲ ﺗﺮى ﯾﺠﯿﺐ ﻟﻚ اﻟﺴﺘﺮﻛﺸﺮ و ﺗﺤﺘﮫ اﺳﻤﮫ‬

‫و ﯾﺴﺄﻟﻚ اﻧﻮ اﯾﺶ اﻟﻠﻲ ﯾﺨﻠﯿﮫ ﻛﺬا ﻛﺬا ﯾﻌﻨﻲ ﺗﻜﻮن ﻛﻞ اﻻﺟﻮﺑﺔ ﻛﯿﻤﯿﺎﺋﯿﺔ‬
‫ ﻗﻮﻟﻚ ﻓﯿﻤﯿﻞ ﻋﻨﺪھﺎ‬UTI ‫ وﻟﻤﺎ اﻟﻔﺎرﻣﺴﺴﺖ ﻋﻤﻞ‬cheak ‫ﻋﻠﻰ اﻟﺒﺮﺳﻜﺮﺑﺸﻦ ﻟﻘﻰ اﻧﻮ اﻟﺒﯿﺸﻨﺖ ﻋﻨﺪھﺎ ﻣﺸﻜﻠﺔ ﻣﺮﺿﯿﺔ ﻣﻊ اﻟﺪواء اﻟﻠﻲ‬
‫اﻟﺪﻛﺘﻮر ﻛﺎﺗﺒﮫ ف اﯾﺶ اﻓﻀﻞ دواء ﻟﮭﺎ‬
‫ وﻛﺎﻧﺖ اﻟﺨﯿﺎرات‬cephalaxin ‫ و‬bactrim ‫ و‬nitrofurantoin ‫ و‬ciprofloxacin
‫و ﻣﻮ ﻛﺎﺗﺐ اﯾﺶ اﻟﻤﺸﻜﻠﺔ اﻟﻠﻲ ﻋﻨﺪھﺎ وﻻ اﯾﺶ اﻟﻠﻲ اﻟﺪﻛﺘﻮر ﻛﺘﺒﮫ ﻟﮭﺎ‬
‫ﺟﺎء ﻛﻤﺎن ﺳﺆال ﻓﻲ دﻛﺘﻮر ﯾﺒﻐﻰ ﯾﺸﻮف اﻻﻧﺘﺮاﻛﺸﻦ ﺑﯿﻦ اﻟﻮراﻓﺎرﯾﻦ و اﻟﮭﯿﺮﺑﺎل ف اﯾﺶ اﻓﻀﻞ ﻣﺮﺟﻊ‬
‫=و اﻻﺟﻮﺑﺔ اﻟﻠﻲ ﺟﺎﺗﻨﻲ ﻛﻠﮭﺎ ادوﯾﺔ ‪ finasteride‬ﺟﺎء ﺳﺆال ﻣﻦ اﻟﺘﺠﻤﯿﻌﺎت اﯾﺶ اﻟﺪواء اﻟﻠﻲ ﻣﺎﺗﻤﺴﻜﮫ اﻟﺤﺎﻣﻞ ﻛﺎن ﺑﺎﻟﺘﺠﻤﯿﻌﺎت اﻟﺠﻮاب‬
‫=‬
‫اﻟﻠﻲ ﺗﺴﺘﺨﺪﻣﯿﮭﺎ ‪ key-word‬اء ﻛﻤﺎن ﻛﯿﺲ و ﯾﻘﻮﻟﻚ ﻟﻤﺎ ﺗﺒﺤﺜﻲ اﯾﺶ ال‬
‫ﻧﺴﯿﺖ ھﻮا ﯾﺒﻐﻰ اﻟﺴﯿﺮب وﻻ اﻟﺴﺴﺒﻨﺸﻦ ‪ phenytoin‬ﺟﺎء ﻛﻤﺎن اﯾﺶ اﻟﻤﺎﻛﺴﯿﻤﻢ دوز ﻣﻦ ال‬
‫‪ protein 97%‬اﻟﻮارﻓﺮﯾﻦ ارﺗﺒﺎطﮫ ﺑﺎل‬
‫‪ volum of distribution‬ف ﻛﻢ ال‬
‫ﺑﺲ ﻛﺬا ﻣﻮ ﺟﺎﯾﺐ ﺣﺎﺟﺔ ﺛﺎﻧﯿﺔ ف ﻣﺎادري ﻟﮫ ﻗﺎﻧﻮن وﻻ‬

‫وﻛﺎﻧﺖ اﻻﺧﺘﯿﺎري ﻣﻦ ‪ 5-3‬او ‪ 50-40‬و ﻛﺬا‬

‫‪Patient use warfarin 5.5 , INR = 9.1 , No bleeding‬‬

‫وﻗﻔﻲ اﻟﻮارﻓﺮﯾﻦ و ﺗﺤﻮﻟﯿﮭﺎ ﻋﻠﻰ رﯾﻔﺎروﻛﺴﺒﺎن‬


‫ﺗﻘﻠﻠﻲ اﻟﺪوز ‪2-‬‬
‫ﺗﻮﻗﻔﻲ و ﺗﻌﻄﯿﮭﺎ اﻧﺘﻲ دوت ‪3-‬‬
‫ﺗﺨﻠﯿﮭﺎ ﺗﻜﻤﻞ و ﺗﺴﻮي ﻟﮭﺎ ﻣﻮﻧﺘﯿﺮوﻧﻖ ﺑﻌﺪ ﯾﻮﻣﯿﻦ )) اﻟﺼﺢ اﻟﻤﻔﺮوض ﻧﻮﻗﻒ ﻛﻢ دوز و ﺑﻌﺪﯾﻦ ﻟﻤﺎ ﯾﺼﯿﺮ ﻧﻮرﻣﺎل ﻧﺮﺟﻌﮭﺎ ﻋﻠﻰ ‪4 -‬‬
‫اﻟﻮراﻓﺎرﯾﻦ ﺑﻌﺪ ﺗﻘﻠﯿﻞ اﻟﺪوز < ﺑﺲ ﻣﺶ ﻣﻮﺟﻮد اﻟﺨﯿﺎر‬
‫ﺟﺎت اﻧﻮ ﻟﻤﺎ ﻛﻠﯿﺔ اﻟﺼﯿﺪﻟﺔ ﺗﺴﻮي ﺣﺎﺟﺔ ﺗﻮﻋﻮﯾﺔ اﯾﺶ ﻧﻮع اﻟﺒﺮﯾﻔﯿﻨﺸﻦ ذَا‬
‫اﯾﺶ ﯾﺼﯿﺮ ﻓﯿﮫ ﺑﺎﻟﺤﺎﻣﻞ ‪ volume of distribution‬ال‬
‫اﯾﺶ ﯾﺼﯿﺮ ﻓﯿﮫ ﺑﺎل ‪ volume of distribution‬ال‬
‫‪ adult‬ﻣﻘﺎرﻧﺔ ب ‪premature infants‬‬

‫ﻛﯿﺲ اﻧﻮ ﺑﻨﺖ ﺻﻐﯿﺮة ﺗﺼﯿﺮ ﻟﮭﺎ ﻧﻮﺑﺎت ﺻﺮع ﻣﺮة ﻛﺜﯿﺮ و ھﯿﺎ ﻣﺎﺷﯿﺔ ﻋﻠﻲ‬
‫و اﻟﺪﻛﺘﻮر زود ﻟﮭﺎ اﻟﺠﺮﻋﺔ ‪valporic acid‬‬
‫ف اﯾﺶ اﻟﺘﺎرﻗﺖ ﻋﻨﺪي اﻧﻨﺎ ﻧﻮﺻﻞ ‪ ٪0‬وﻻ ‪ ٪25‬وﻻ ‪ ٪50‬وﻻ اﻧﻮ اﻟﻨﻮﺑﺔ ﺗﺠﻲ ﻟﮭﺎ ﻣﺮة ﺑﺎﻻﺳﺒﻮع‬
‫ا ﻛﻤﺎن اﻧﻮ ﻓﻲ ﻧﯿﺮس طﻠﺒﺖ دواء ﻣﻦ اﻟﺼﯿﺪﻟﯿﺔ وﻛﺎن اﻻوردر ﺳﺘﺎت وﻗﻠﮭﺎ اﻟﻔﺎرﻣﺴﺴﺖ اﻧﻮ ﺗﺮﺟﻊ ﺗﺄﺧﺬو ﺑﻌﺪ رﺑﻊ ﺳﺎﻋﺔ‬
‫ﻟﻤﺎ رﺟﻌﺖ ﻟﻘﯿﺖ ﻓﺎرﻣﺴﺴﺖ ﺛﺎﻧﻲ و ﻗﺎل ﻟﮭﺎ اﻧﻮ ﻣﺎﯾﻌﺮف ﻋﻦ ذَا اﻻوردر واﻧﮭﺎ ﺗﺠﯿﺐ ﻟﮫ اﻻوردر ﻣﺮة ﺛﺎﻧﯿﺔ ف أﺗﺄﺧﺮ ﺗﺤﻀﯿﺮ اﻻوردر‬
‫‪ 90‬دﻗﯿﻘﺔ‬

‫و ﻛﺎن اﻟﺴﺆال اﻧﻮ اﯾﺶ ﺳﺒﺐ ذي اﻟﻤﺸﻜﻠﺔ‬

‫ﻓﻲ اﻟﻔﺎرﻣﺴﻲ ﺳﺘﺎف ‪ poor communication‬اﻧﺎ ﺣﻄﯿﺖ اﻧﻮ‬

‫اﻟﺜﯿﺮاﺑﻲ ‪ update‬ﺟﺎء ﻛﻤﺎن دﻛﺘﻮر اﻣﺮاض ﻗﻠﺐ ﺳﺄﻟﻚ ﯾﺒﻐﻰ اﻓﻀﻞ ﻣﻜﺎن ﯾﻼﻗﻲ ﻓﯿﮫ‬

‫‪ guideline‬اﻧﺎ ﺣﻄﯿﺖ ال‬

‫اﯾﺶ ﻧﻌﻄﯿﮫ ؟ ‪ status asthma‬ﺑﯿﺸﻨﺖ ﻋﻨﺪو‬


Type stuructue :
‫ ﯾﻜﻮن اﻟﺠﻮاب‬Disterioisomer
phase that conduct the unexpected adver drug reaction
-Phase 4 (post markting)
3-infection phase prodrome defintion
-early symptom that signals the onset of an illness or disease
4- om salm sale herbal medcin for dm and pepole complaine
-inspection
7-emulsion
-emulsion is a mixture of two or more liquids that are normally immiscible

suspsion defintion
-suspension is a liquid with small pieces of drug. The drug is not complete dissolved in the solution.
8- morbidty defintion
-Refers to disease states
9- ‫ ﺟﺎء ﻛﺜﯿﺮ ﻣﻦ‬cost ‫ اﺳﺌﻠﺔ‬10 ‫ﺗﻘﺮﯾﺒﺎ‬
10 - toxidrome defintion
-syndrome caused by a dangerous level of toxins in the body.
11- minor injury with unknow history TD
12 deep injury with unknow history
TIG +TD
13 - patinet eats penute and have swelling face deficult to breath which type of reaction ?
- Hypersensvtie
14- in hypersnsvite which appers ?
- Histamin
15-non esstional amino acid
Cysteine
16 - vit k adminstration route for infant
-Intramuscular (IM)
17 - look alike soud alike solution ?
- Tall man letter
18 -‫ ﺟﺎء ﻛﺜﯿﺮ ﻣﻦ‬Ig ‫ ﻣﺜﻞ‬which one cross the placenat
19 - have gamma chain ? IgG
20- which one appers in hypersensvti ? Ige
24- T - lymphod ‫ﺗﺘﺼﻨﻊ وﯾﻦ‬
-migrate to the thymus gland in the neck, where they mature and differentiate into various types of
mature T cells and spleen
25 - what hormone secreted from antior pituitory gland

- The anterior pituitary gland secretes 7 hormones: follicle -stimulating hormone, luteinizing
horomone, adrenocorticotropic horomone, thyroid -stimulating horomone, prolactin, endorphins, and
growth hormone
26-clostirduim ? Gram + Rod anarobic
- Clostridium difficile is a Gram-positive, spore-forming obligate anaerobe
27- ‫ ﺗﻌﺮﯾﻒ‬catacory c for preganat
-Animal reproduction studies have shown an adverse effect on the fetus and there are no adequate and
well-controlled studies in humans,
28- p value ?
A- > signficant clinical
B- < signficant statical ▫
C- >signficant statical
D - < signficant clinical
29-‫ ﺟﺎء ﻋﻦ‬scfhs ‫ھﻲ اﻟﻠﻲ ﺗﻌﻄﻲ اﻟﺮﺧﺺ ﻟﻠﻤﮭﻨﮫ‬
30 - drug inhibtor prolactin
31- woman with hypertension what is proper oral contrrceptive for her
32- carbidopa function in levidopa drug ?
- decarboxylase inhibitors.
33- which drug when we give we montering the heart 6 houre later ?‫ اﺧﺘﺮت‬fingolimed
34- NISAD ACTION ON STOMACH ?
- blocking the Cox-1 enzyme and disrupting the production of prostaglandins in the stomach,
35- captopril SE ?
-Defect in test
36- captopril SE ?
- DRY COUGH
37- Direct inhibtor of thrombin ?
-Dabigatran
38 -chlamydia treatment ? -Azithromycin

40 -warfaren was add ob levoflocxine for Respirtory infection ?


Drug -drug interaction▫
drug - food interaction
41- metoformine SE ?
lactic acidosis
42- to treat patient with tb what is the bast way ?
- DOT DIRCTe observation treatment

46- heparin antidote


Protamin
47-benzodiazpen antidote
Flumazenil

48- which antibiotic cause c.deffic ? clindmycin


49- ethamutaol TB Drug SE
Effect eyes vision
50- dispens of diltazam for ? one month
51- isotonic solution ? Ringer
52- anemia low MCA and low hemoglobin which type ?
Iron deficiency
53- which one of the TB drug interaction with most of the drugs ?
Rifampen
54-warfarin test ?
INR
55- ‫ ﺟﺎء ﻛﯿﺲ ﻣﺮﯾﺾ ﯾﺎﺧﺬ‬codien ‫ ﺟﺎء ﻟﻠﻄﻮارى ﯾﺸﺘﻜﻲ ﻣﻦ‬tchycardia palpation what the causa ‫؟‬
Rapid metoblar of c2d6
56- drug interact with st john wort ? ‫ ﻣﻦ اﻻﺧﺘﯿﺎرات دواء‬setralin (SSRI)
57- case patient 3 y.o with fever and cough with no tonsilit enalargment the docter said it could be
viral infection what give ?
a- paractamol
b- osaltamivir
58- pregnant with HIV what treament ?
zidovudine
59- trament of malignant hypertenstion ?
-nitroprusside sodium
60- ‫ ﺟﺎب ﻛﯿﺲ ﻣﺮاه ﻣﻌﺎھﺎ اﻛﺘﺌﺎب وﺳﻤﻨﺖ وش اﻟﺴﺒﺐ او اﻟﺪواء اﻟﻤﺴﺒﺐ ؟‬bupropion ‫اﺧﺘﺮﺗﮫ‬
61 - ‫ ﺟﺘﻨﻲ اﻟﺼﻮره ﺣﻘﺖ‬ACE ‫ و‬ARB ‫ و‬Rennin
62- antiplatal cauase dypensia
ticagrelor
63 - ‫ ﻣﺮﯾﺾ ﺟﺎء ﻟﻠﺼﯿﺪﻟﯿﮫ ﻣﺎﻛﺎن ﯾﻘﺪر ﯾﺘﻜﻠﻢ ﻣﻦ اﻟﻜﺤﮫ وﻛﺎن ﯾﺎﺧﺬ دواء ﺿﻐﻂ‬linsopril ‫؟‬
‫ ﻧﺴﻮي‬refill to doctor bec maybe the dry cough from the ACEI
64- WHICH one is correct about vit D ?
‫ اﺧﺘﺮت‬D2 and D3 inactive
66- treatment vomtiong in preganat ?
B6
69- ‫ﻣﺮﯾﺾ ﺗﺠﯿﮫ اﻻﻧﻔﻠﻮﻧﺰا داﯾﻢ وش ﻧﻌﻄﯿﮫ ﻋﺸﺎن ﻧﻤﻨﻌﮭﺎ او وش ﯾﺎﺧﺬ ﻋﺸﺎن ﯾﻤﻨﻌﮭﺎ ؟‬
influanza vaccine anually ‫ﺳﻨﻮﯾﺎ‬
71 - ‫ ﻣﺮﯾﻀﮫ ﻋﻨﺪھﺎ‬HIV ‫( وﺗﺒﻲ ﺗﺎﺧﺬ ﺗﻄﻌﯿﻢ‬vaccine )
- no vaccine for her becauss she have crtical illnss
72- Rate control treatment ?
Bb
75- ‫ ﻣﺮﯾﻀﮫ ﻋﻨﺪھﺎ‬ostoprosis and hypotherod ‫وﺟﺎﯾﮫ ﺗﺎﺧﺬ ادوﯾﮫ ﺳﻜﺮ وﻻﺣﻆ اﻟﺼﯿﺪﻟﻲ ھﺬا واﻗﺮت اﻧﮭﺎ ﻣﺎﺗﺎﺧﺬ وﻻ دواء واﻧﮭﺎ‬
‫ﻣﻠﺘﺰﻣﮫ ﺑﺎدوﯾﺘﮭﺎ‬
A- refill to doctor untreat▫ patinet
B- NON adherens
C - do nothing
76- ‫ ام ﺣﺎﻣﻞ وﻣﻌﮭﺎ‬Hiptits B ‫وﻟﺪت وش ﻧﻌﻄﻲ اﻟﻤﻮﻟﻮد ﺗﻄﻌﯿﻢ ؟‬
Vaccine hib and immubglobine

78 - xalant ‫اﺗﻮﻗﻊ ﻛﺬا اﺳﻢ اﻟﺪواء وﻛﺎﺗﺐ وش ﯾﻌﺎﻟﺞ ؟‬


Glaucoma
79 - high intnstiy stant
ROS 20
81- samvastatin best time take it ? At bedtime
82- ‫ وﺑﺮﺿﻮا ﺟﺎﺗﻨﻲ ﻛﯿﺲ وﻓﯿﮭﺎ ﺗﺤﺎﻟﯿﻞ اﻻ ﺗﺤﻠﯿﻞ‬phenytoin ‫ﻣﻮ طﺎﻟﻊ‬
‫ وﻣﻌﻄﯿﻨﻲ ﺑﺎﻟﺘﺤﺎﻟﯿﻞ‬albumin ‫ ﻧﺎزل ﯾﻌﻨﻲ وش اﺳﺘﻨﺘﺞ ؟‬20 ‫ﻗﻠﯿﻞ‬
A- phenytoin increase in plasma becuses decrese albumin ▫
B- phenytoin decres becuses the decress of albumin

ow can coronavir trans :


‫ ﺣﻄﯿﺖ‬respirator
‫واﻟﺨﯿﺎرات ﻛﻮﻧﺘﺎك وذ ﻓﻮد واﻟﺒﺎﻗﻲ ﻧﺴﯿﺖ‬

❇limited vaccine for infants


pneumococcal
rota ‫ﺣﻄﯿﺖ‬
bcg
mmr
❇amlodipine and warfarin CI plant not use:
grapefruit
❇Structure drug ppi
❇2 Struc to phynatoin /add OH mining :
Ox or reduc or carbox or..
❇drug toxic metabolic acidosis:
spironolactone
❇Patient came with greenish yellow sputum which drug to give? Guaifenesin
❇Antiviral to treat H1N1 ? oseltamivir

‫ﺳﻮال ﯾﺸﺎﺑﮭﮫ ﺑﺎﻟﻔﻜﺮه‬Which one carry high risk for IV admixture?
Preparing IV admixture from non-sterile products

The synthesis of glycogen from glucose called?
Glycogenesis

‫ﺳﻮال ﯾﺸﺎﺑﮭﮫ‬What is apoptosis ? programmed cell death
❇One came to you in the pharmacy and told you she opened the eye drop 45 days ago what should she
do?
Discard the medication ( expiry date 28 days after open)

High intensity statin ?
Rosuvastatin 20
❇midazolam antidote : Flumazenil
❇Antimalarial medicines : Quinine
❇Second drug mlaria :Artesunate ‫اﺧﺘﺮت‬
❇Which TB drug most likely to interact with HIV drug : Rifampicin

Tamoxifen ? selective estrogen receptor modif

ferrous sulfate or ferrous gluconate ‫ واﺣﺪ ﻣﻨﮭﻢ اﻟﻤﮭﻢ ﯾﺴﺒﺐ‬Black or dark stools
❇Iron is stored in the liver : ferritin
❇Sickle cell pain crises ? Morphine
❇PATIENT TAKE VANCOMYCIN AND GOT RED MAN SYNDROME WHAT TO DO ? stop
immediately
❇WHAT IS THE FIRST CHOICE WITH SYMPATHETIC SYMPTOMS OS SNAKE BITE Antidote
?
Antivenom‫ﻧﻌﻄﯿﮫ اﻧﺘﻲ دوت ﻟﻠﻌﻀﮫ ﻣﺸﺎﺑﮫ ﻟﮫ اﻟﻤﮭﻢ ﻣﺎﻧﻌﻄﯿﮫ ﺷﺎرﻛﻮل‬
❇TTT of pregnant with UTI ? nitrofurantoin
❇Prazocin MOA ?
selective alpha 1-adrenergic blocking antagonist
❇CHADS2 vasc score ‫اﺣﺴﺐ‬
❇ Which help with addiction to codeine ? Methadone
❇ What is monitor clozapine ? ANC
❇ first-or zero order kinetics
‫ﺟﺎب ﺗﻌﺮﯾﻒ اﻟﺪوس دﺑﻨﺪﻧﺖ وﻗﺎل اﯾﮭﻢ ﯾﺘﺒﻌﮫ زﯾﺮو او ﻓﯿﺮﺳﺖ‬
❇ The standard treatment of H. Pylori? Clarithromycin, Amoxicillin,PPI
❇ Garlic and warfarin interaction ?
Increased warfarin effect
❇ treatments dose start enoxaparin?
Enoxaparin 40 mg sc once daily‫🤷اﺧﺘﺮﺗﮫ‬
1‫ﻣﺎﻛﺎن ﻓﯿﮫ‬mg
❇ A student print artical.. which ?
Pubmed🤷
❇ complications of liver cirrhosis: Ascites
❇ Patient is use Metformin + Glibenclamid, what is the most appropriate monitoring for him?Liver
function test
❇ Empty morphine vials are discarded after: 6 months
❇ The drug in the market 2009 when expire ? After 20 years2029

High affinity picture A more high B affinity


L glu ti D glu
Pharmacogenom
‫‪ TPN :‬ﺳﺄل اﯾﺶ اﻟﺤﺎﻟﮫ اﻟﻠﻲ ﻻزم ﻧﻌﻄﯿﮭﺎ‬
‫‪Brain hemorrhage‬‬
‫‪Gastric ..‬‬
‫وراﺑﻊ ﻧﺴﯿﺘﮫ ‪Bowel cancer‬‬
‫‪ spironolactone‬ﺟﺎب ﻛﯿﺲ وﻛﺎﺗﺐ ادوﯾﺘﮫ وﺟﺎب اﻟﺘﺤﺎﻟﯿﻞ ﻣﻦ ﺿﻤﻨﮭﺎ اﻟﺒﻮﺗﺎﺳﯿﻮم ﺷﻮي ﻋﺎﻟﻲ وﺳﺄل ﻣﯿﻦ ﻣﻤﻜﻦ ﯾﻜﻮن ﺳﺒﺒﮫ ؟‪-‬‬
‫ﺟﺮﻋﺔ اﻟﻜﺎﻟﺴﯿﻮم ؟ ‪-1200‬‬
‫‪ patient weight‬ام ﺟﺎت اﻟﺼﯿﺪﻟﯿﮫ ﺗﺒﻐﻰ ﻣﺴﻜﻦ ﻟﻮﻟﺪھﺎ ‪،‬اﯾﺶ اﻟﻤﻌﻠﻮﻣﺎت اﻟﻠﻲ اﻟﻤﻔﺮوض ﻧﺴﺄل ﻋﻨﮭﺎ ؟اﺧﺘﺮت‪-‬‬
‫؟ ‪ ATP‬اﻟﺠﻠﻮﻛﻮز اﻟﻮﺣﺪه ﻛﻢ ﺗﻌﻄﻲ‪-‬‬
‫‪12،20،48‬‬
‫ﻣﺮﯾﻀﮫ ﻗﺮرت ﺗﻮﻗﻒ ادوﯾﺘﮭﺎ ﺑﻌﺪ ﻣﺎﻧﻘﻠﺖ اﻟﺒﯿﺖ اﻟﺠﺪﯾﺪ وﺑﻌﺪﯾﻦ ﺷﺎﻓﻮھﺎ ﺗﻠﻮم زوﺟﮭﺎ ﻻﻧﮫ ﻣﺎﻣﻨﻌﮭﺎ ﻋﻦ ﻗﺮارھﺎ اﯾﺶ اﻓﻀﻞ ﺗﺼﺮف ‪-‬‬
‫ﺑﮭﺎﻟﺤﺎﻟﮫ ؟‬
‫اﻟﺨﯿﺎرات ﻛﺎﻧﺖ ﻧﺠﺒﺮ زوﺟﮭﺎ ﯾﺠﯿﺒﮭﺎ ﻣﻮاﻋﯿﺪه ‪،‬ﻧﺒﺤﺚ ﻋﻦ اﻟﻤﺸﺎﻛﻞ اﻻﺟﺘﻤﺎﻋﯿﮫ اﻟﻤﺘﻌﻠﻘﮫ ﺑﺎﻟﻨﺘﺎﺋﺞ ‪ ،‬او اﻟﻤﺸﺎﻛﻞ اﻟﺒﯿﻮﻟﻮﺟﯿﮫ وﺧﯿﺎر ﺛﺎﻟﺚ‬
‫ﻧﺴﯿﺘﮫ‬
‫‪، ppsv‬ﺑﻌﺪﯾﻦ ﻛﯿﻒ ﻧﺤﻤﻲ ﻣﻨﮫ ؟ اﻟﺠﻮاب ﻓﺎﻛﺴﯿﻦ ‪ meningitis‬ﻣﺮﯾﻀﮫ ﺟﺎھﺎ‪-‬‬
‫وﺑﺘﺎﺧﺬ ﻻﯾﻒ ﻓﺎﻛﺴﯿﻦ ﻧﻌﻄﯿﮭﺎ واﻻ ﻻ وﻟﯿﺶ؟ ‪ Iv Acyclovir‬ﻣﺮﯾﻀﮫ اﺧﺬت‪-‬‬
‫ﺣﺴﺎﺑﺎت ﻟﻠﻜﺮﯾﺎﺗﯿﻨﯿﻦ ﺗﻘﺮﯾﺒﺎ ً ﺟﺎﺗﻨﻲ ﺳﺘﮫ او ﺳﺒﻌﮫ ﺗﻌﻮﯾﺾ ﻣﺒﺎﺷﺮ‪-‬‬
‫اﯾﺶ ھﻮ ؟ ‪ A‬ﺟﺪول اﻷدوﯾﮫ ﺟﺎ ﺑﺮﺿﻮ ‪،‬ﯾﺠﯿﺐ اﻟﺠﺪول ودواء‪-‬‬
‫‪-H.pylori treatment‬‬
‫‪ ،‬اﯾﺶ اﻟﺨﻄﺄ اﻟﻠﻲ ﺳﻮاه ‪ documents‬ﻣﻊ زوﺟﺘﮫ واﻟﺼﯿﺪﻟﻲ ﻛﺘﺐ ‪ diazepam‬ﻣﺮﯾﺾ ﻧﻔﺴﻲ ﺟﺎ اﻟﺼﯿﺪﻟﯿﮫ وﻗﺎل ﻟﻠﺼﯿﺪﻟﻲ اﻧﻮ ادﻣﻦ‪-‬‬
‫اﻟﺼﯿﺪﻟﻲ ؟‬
‫! ﺟﺎب ﻛﯿﺲ ﻣﺮﯾﺾ ﻋﻨﺪه ﻋﺪوى ووﺻﻒ ﻛﺎﻣﻞ ﺑﻌﺪﯾﻦ اﻟﺴﺆال اﯾﺶ اﻟﺸﺠﺮه اﻟﻠﻲ اﺧﺬﻧﺎ ﻣﻨﮭﺎ ﻋﻼج اﻟﺤﺎﻟﮫ‪-‬‬
‫‪ benzodiazepines‬ﺳﺘﺮﻛﺸﺮ‪-‬‬
‫ف ﻋﻼج ﻧﻮع ﻣﻌﯿﻦ ﻣﻦ اﻟﺴﺮطﺎن ﻟﻤﺎ ﺑﺤﺚ اﻟﺼﯿﺪﻟﻲ ﻟﻘﻰ ﺣﺎﻟﮫ وﺣﺪه ‪ monoclonal antibody‬دﻛﺘﻮر اﺗﺼﻞ ع اﻟﺼﯿﺪﻟﻲ وﺳﺄﻟﮫ ﻋﻦ‪-‬‬
‫ﻓﻘﻂ ﺗﺨﺺ اﻟﻤﻮﺿﻮع اﯾﺶ اﻟﻤﻔﺮوض ﯾﺴﻮي ؟‬
‫‪ iv omeprazol‬اﯾﺶ ﻧﻌﻄﯿﮫ ؟ ‪duodenum‬ﻣﺮﯾﺾ ﻋﻨﺪه اﻟﻢ ﺷﺪﯾﺪ ف اﻟﺒﻄﻦ وﺑﺎﻷﺷﻌﮫ طﻠﻊ ﻋﻨﺪه ﺗﻘﺮح ب‪-‬‬
‫‪1. Bentonite is added as:‬‬
‫‪A) Suspending agent‬‬
‫‪B) Vehicle‬‬
‫‪C) Preservative‬‬
‫‪D) Emulsfying agent‬‬

‫‪2. MOA of Dipyridmole:‬‬


‫)‪( I don’t remember the choices‬‬

‫)‪3. UTI in pregnant ( Many questions‬‬

‫‪4. Enoxaparin treatment dose‬‬

‫‪5. MOA of Amphetamine‬‬

‫‪6. Status Asthmatics treatment‬‬

‫‪7. Plant for Burn ➡ Aloe vera‬‬

‫‪8. MOA of Losartan‬‬

‫‪9. Definition of Management :‬‬


‫‪- The art of increasing Profitability‬‬
‫‪- The art of increasing Possibilities‬‬
‫‪- The art of increasing Productivity‬‬
10. ADRs reported to ? SFDA

11. Vaccination are regulated from? MOH

12. Phase 2 Enzymes are mainly focused in which part?


- Endoplasmic reticulum
- Nucleus
- Cytoplasm

13. Color Label of Hazardous medications?

14. Many questions about cases that have an allergy from penicillin so wha are the alternatives?

15. The structure of penicillin allergy the one with “S”

16. Structure of Ciprofloxacin asking what the F in the structure indicates?


- Increase the beta-lactamase resistance
- Decrease activity toward gram -ve bacteria
- Increase lipophilicity to cross over bacterial cell wall.

17. Structure of Malonate or something asking that the energy is 30 So what is the structure feature?
- Exoergic
- Endoergic
- Heat producing 🤷

18. A women don’t want to take the medication for her osteoporosis and osteoarthritis but she’s taking
other medications for HTN and DM, and when the pharmacist asked why? She said that it cause
diarrhea so she stopped it 🙂 the question is: what is this action called?
- Nonadherence
- Untreated condition
- ADRs

19. A long story about a pharmacist in an interview and the interviewer just owned a new pharmacy
and give him a case to work on and make a projection on how to manage the income? So what
pharmacist should take into consideration?
- Cash of income and outcome
- Number of pilgrims in the last 5 years
- Total budget or something (I choosed this one) ً ‫ﻣﻦ اﻟﻜﯿﺲ طﺒﻌﺎ‬

20. Phase 2 in metabolism? Conjugation

21. A case with HTN patient complains about non-productive cough and want a solution for his
problem?
- Guanifacin
- Dextromtherophan 15 mg BID
- Dextromethrophan 30 mg BID
- Home remedies (Honey)

22. Intangible cost ➡ fatigue - pain


23. 795 USP ➡ non-sterile

24. 797 USP ➡ Sterile

25. In CUA ➡ QALY

26. In CBA ➡ Human Captial

27. The circle of Antibiotics with bacterial resistance or sensitivity

28. Essential Fatty acid ➡ Lineolic acid

29. Which of the following has high estrogen and is contraindicated with a women with stroke?
- vaginal ring
- Progestin
- IUD
- Transdermal patch

30. A case with gout asking what is the drug that come from this source (Autumun - crocus)
The answer is: colchicine

31. You should now more about Hypotonic , Hypertonic and isotonic solution (Many Qs)

32. Which of the following Decreases warfarin activity?


- Cimitdine
- Rifampcin
- St.John warts

33. QRS ➡ ventricular depolarization

34. PR interval ➡ Atrioventricular conduction

35. Example of tertiary resources? ➡ Review articles

36. Patient with his son coming from outside the city for a treatment the cost for the hotel for his son is
considered? ➡ Direct nonmedical

37. A study with prevalence ➡ Cross-sectional

38. A study about the effect of contraceptives upon women and follow up overtime so starting with risk
factor first so ➡ Prospective cohort

39. The lowest number in (Patients) in clinical trial phases are within which phase?
I choose 2 because phase 1 include healthy individuals not patients 🤷

40. First order with fixed dose and regular interval and then asked what will affect the time we will
need to reach the Steady state?
- Dose of the drug
- Frequency of the drug
- Half life of elimination

41. Cancerous patient what Anticoagulant is suitable for him? LMWH

42. Rate and Rhythm control (Many questions)

43. Egg allergy long case ➡ not to give MMR

44. Deep and minor wounds cases.

45. A case of child with a flu and needing to reduce the duration of the flu ➡ Vitamin C

46. The curve with 2 drugs and which one is more effective and more potent➡ the drugs with taller
and near zero.

47. Heparin Antidote ➡ Protamine sulfate

48. Carpamezapine ➡ HLA-B1502

49. Allopurinol ➡ Steven-johnsons

50. Dose of Allopurinol in renal impaired patient

51. Effect of smoking on contraceptives ? 100 mg

52. Calculating the needed tablets from a suspension (‫)زي اﻟﻠﻲ ﻓﻲ اﻟﺰﺗﻮﻧﺔ‬

53. BMI - CrCl for male and Female , VD ‫واﻏﻠﺐ اﻟﺒﺎﻗﻲ طﺮﻓﯿﻦ ﻓﻲ وﺳﻄﯿﻦ ﻣﺎﻋﺪا ﻛﻢ ﻣﺴﺄﻟﺔ ﻣﺎﻓﮭﻤﺖ اﻟﻤﻄﻠﻮب وﺣﺎوﻟﺖ‬
‫🤝 ﻟﯿﻦ ﺧﻠﯿﺘﮭﺎ ﺻﺪﻗﺔ ﺑﺮﺿﻮ‬

54. Structures of epimers and isomers

55. Septic shock patient with a normal Na and Cl and the question was to add?
- Dopamin
- Normal saline
- Ringer solution

56. Brown urine ➡ Nitrofurantoin

57. A 69 male having erectile dysfunction and doesn’t want to take pills or injection?
- vacuum device
- Testosterone gel
- Sildanifil pills
- Psychotherapy

58. A female patient doesn’t want to take insulin because it causes weight gain so she stopped it.. what
to do ?
- Refer her to the physician
- Change her dose
- Change her beliefs about insulin
- Tell her yes it does 🙂

58. Patient come to hosptial with lab results of High AST and ALT but normal bilirubin & Amylase:
- Hepatic Cirrhosis
- Hepatic Stenosis
- Pancreatitis

59. Hepatic encephalopathy what to check?


- Albumin
- Ammonia

60. Malarial treatment in Saudi Arabia?


Artesunate - Sulfadoxine - Pyrimethamine ‫ﺗﺠﻲ ﻛﻠﮭﺎ ﻣﻊ ﺑﻌﺾ ﻓﻲ اﺧﺘﯿﺎر واﺣﺪ‬

61. Patient was suspected to have toxicity from opioids what to give him?
- Naletrexone 400 mg IM
- Naletrexone 400 mg Orally
- Flumazenil

62. Anti-estrogen that treats breast cancer ➡ Tamoxifen

63. Phenytoin corrected level

64. Stop clopidogrel before surgery ➡ 5 days

65. Stop UFH before surgery ➡ 4-6 hours ‫ ﻣﺘﻰ ﻧﺴﻮي اﻟﻌﻤﻠﯿﺔ؟‬٢ ‫ ﯾﻘﻠﻜﻢ وﻗﻔﻨﺎه اﻟﺴﺎﻋﺔ‬.. ‫ﺑﺲ ھﻨﺎ ﺗﺤﺴﺒﻮھﺎ‬
٨ ‫ ﯾﻌﻨﻲ ﯾﻜﻮن اﻟﺴﺎﻋﮫ‬/ ‫ ﺳﺎﻋﺎت‬٦ ‫ﺑﻌﺪ‬

66. What to give from these to type 1 DM:


- Glyburide
- Pramlintide
- Sitagliptine
- ‫وﻣﺎﻓﻲ ﻣﯿﺘﻔﻮرﻣﯿﻦ‬

67. Long case asking what’s the best Antiarrythmic for HF? ➡ Amiodarone

68. Diabetes inspidus ➡ Desmopressin ‫ ﻣﺮات‬٣ ‫أﺗﻜﺮر‬

69. A long case with patients having QT prolongation and asking what antibiotic we should avoid? ➡
Azithromycin (Macrolid)

70. Long story ‫ اﻟﺰﺑﺪة‬they want to know what is the unknown drug by showing the results of some tests
and the only postive test was Dragendroff test so the answer ➡ Alkaloid

1. Case of patient with seizure (I think also diabetes), now the patient newly diagnosed with
depression and ask about the best management in this case? The answer includes Buspirone, fluoxetine,
fluvoxamine, mirtazapine.
2. Case of patient with diabetes and now the patient newly diagnosed with depression and ask
about the best management in this case? The answer includes duloxetine
3. Case of patient in the emergency with (I think gastrointestinal proliferation but I am not sure)
and ask about the management? The answer includes this options omeprazole IV infusion, enoxaparin,
sucralfate IV infusion.
4. Immunoglobulin in the passive immunity?
5. Doxazosin MOA? The answer includes this options Presynaptic beta blocker, Postsynaptic beta
blocker, Presynaptic alpha blocker, Postsynaptic alpha blocker.
6. Counseling of plan B pill contraceptive? The answer includes this option take daily, take
within 72 hours, take within 96 hours.
7. What is the Antibiogram? Susceptibility test of specific microorganism to beater antimicrobial
drug.
8. Dose of calcium in female patient 67 years, and also for female patient 79 years.
9. What increase the myopathy risk when give in combination with gemfibrozil? Statin
10. 57 years old male patient (I think has history of hypertension) and now need drug to control the
lipid (I forgot cholesterol or triglyceride)? The answer includes this options High intensity statin,
Moderate intensity statin, Statin with ezetimibe.
11. 4 years child came to the hospital with diarrhea from 2 days and now diagnosed with rotavirus
and ask about the best management? The answer includes this options Acyclovir, hydration,
loperamide.
12. Male patient has history of hypertensions and angina now came to the hospital with erectile
dysfunction and ask about the best management? The answer includes nonpharmacologic treatment,
also sildenafil and other drugs from this class.
13. Long case with lab results include CrCl less than 30ml/min and ask what the medication must
be hold? The answer includes metformin.
14. Long case of patient came to the hospital with deep wound and unknown history of vaccine?
15. Anther Long case of patient came to the hospital with minor wound and unknown history of
vaccine?
16. Case of doctor ask the pharmacist about who responsible about vaccine in Saudi Arabia?
17. Case of cardiology doctor ask the pharmacist about the most update reference of the therapy of
heart disease?
18. Principal of ethics?
19. Long case and ask about the type of hotel cost?
20. Long case and ask about type of travel cost?
21. Long case of patient takes enoxaparin and develop bleeding and ask about the best
management? The answer includes protamine.
22. Antidote of atenolol.
23. What is the best herbal for triglyceride? I think fish oil
24. The type of Interaction between warfarin and amiodarone? The answer includes drug-drug
interaction, drug food interaction.
25. Long case and ask about drug not used to treat the pregnant women with malaria? The answer
includes doxycycline.
26. Female on warfarin and now pregnant what is the best intervention?
27. Case of patient drink tea for 2 weeks and now develop palpitation and other symptoms and ask
about what this herbal? The answer includes glycosamide, digitals.
28. What the medication from plant source use to treat the malaria? Quinine.
29. Curve and ask about therapeutic index?
30. Calculation of T ½?
31. Calculation of specific grafity?
32. Name of Vitamin C? Ascorbic Acid.
33. Name of Vitamin B12? Cyanocobalamin
34. Management of Raynaud syndrome?
35. Management of hypophosphatemia?
36. Encapsulated bacteria?
37. Treatment of MSSA?
38. Prophylaxis of travel diarrhea?
39. Long case of patient with HF and ask about the anti-arrythmia drug can be use with this case?
40. Case of patient with diabetes and now diagnosed with AF and ask about the best management?
The answer includes amiodaron, digoxin.
41. Pregnant in the first trimester with UTI? The answer includes cefalexin, bacterium,
ciprofloxacin.
42. Barbiturate in elderly cause? Agitation
43. The first vasopressor uses in septic shock? Norepinephrine
44. Patient with septic shock and not respond to IV fluid what the next step? Add norepinephrine
45. Medication recently add to (I think narcotic List)? Pregabalin
46. Folic acid start before pregnancy? One month.
47. Case with elevated AST and ALT and normal bilirubin? Liver cirrhosis
48. Long case with lab results in patient with hypertension and ask about the complication of
hypertension? According to the patient lab results I choose chronic kidney disease.
49. Patient take oxycodone, oxybutynin (and also, I think lisinopril) and ask about expected side
effect?
50. Monitoring of peripheral neuropathy in diabetic patient every?
51. Influenza vaccine in high risk patient? The answer includes annually, every 6 months, every 5
years.
52. Patient take metformin, sitagliptin, pioglitazone and develop liver disease and ask about
medication must be hold? pioglitazone
‫ﺳﺆال ﻋﻦ اﻻﯾﺮون اﻟﺠﻮاب اﻟﻔﯿﺘﺎﻣﯿﻦ ﺳﻲ‬
‫ اﻟﻤﺴﺎﺋﻞ وﺳﻂ اﻏﻠﺒﮭﺎ طﺮﻓﯿﻦ ﻓﻲ وﺳﻄﯿﻦ ﻣﺎﻋﺪا ﻣﺴﺌﻠﺔ ﻋﻦ‬LD and VD
Structure
‫ ﻟﺒﻨﺴﻠﯿﻦ اﻟﺤﺴﺎﺳﯿﺔ ﻧﺨﺘﺎر اﻟﻠﻲ ﻓﯿﮫ ﺣﻠﻘﺔ‬B-lactam
‫ ﺳﺆال ﻋﻦ‬HIV
CD4
CD1
CD2
CD3
‫ وﺑﻌﺪ ﺣﻘﺖ‬ECG ‫اﻟﺮﺳﻤﮫ واﻟﺮﺳﺰﺗﻨﺲ ﺣﻖ اﻟﺒﻜﺘﯿﺮﯾﺎ اﻟﻘﺮص‬
‫? ‪Potency Vs Efficacy‬‬

‫‪If Hight by cm‬‬

‫وﺑﺮﺿﻮا ﺳﺆال وﺣﺪه ﻋﻨﺪ ﻋﯿﻮﻧﮭﺎ اﻧﺘﻔﺎخ ﺑﺴﺒﺐ اي ﻋﻼج اﻟﻠﻲ ھﻮ‬
‫‪ACEI‬‬
‫اﻧﺎ ﺑﻘﻮﻟﮫ ان ام ﺻﺎﻟﺢ ﺗﺒﯿﻊ اﻋﺸﺎب ﻓﻲ اﻻﻧﺴﺘﺎ وﺗﻘﻮل اﻧﮭﺎ ﺗﻌﺎﻟﺞ اﻟﺴﻜﺮ واﻟﻨﺎس ﻣﺪري واﺣﺪ ﺑﻠﻎ او اﻧﮫ ﺻﯿﺪﻟﻲ ﺑﻠﻎ ﻣﻮ ﻣﺘﺬﻛﺮ ﺑﺎﻟﻀﺒﻂ ﺑﻠﻎ‬
‫‪ SFDA‬وش ﻻزم ﺗﺴﻮي ‪SFDA‬‬
‫‪ inspection‬ﺣﻄﯿﺖ ﺟﻮاب‬
‫وﺑﺮﺿﻮا ﺳﺆال ﻣﯿﻦ ﺗﻌﺎدل اﻟﺸﮭﺎدات اﻟﻠﻲ ﺗﺠﻲ ﻣﻦ ﺑﺮا‬
‫اﻟﻮزارة‬
‫‪definition of area under the curve‬‬
‫‪٢- examples of rate controls‬‬
٥- cases of copd
‫ﻛﻠﮭﺎ واﺿﺤﺔ ﺗﺘﻜﻠﻢ ﻋﻦ وش ﻧﻮع اﻟﻜﻼس ؟ وﺗﺘﻜﻠﻢ ﻋﻦ اﻷدوﯾﺔ اﻟﻤﺴﺘﺨﺪﻣﺔ‬.

Fingolimod
‫ ﻧﺎﺳﻲ اﻟﻌﻼج وﷲ‬، ‫ﺑﺎﻟﻨﺴﺒﺔ ﻟﻠﺤﺎﻣﻞ ھﻞ ﺗﺴﺘﻤﺮ او ﺗﻮﻗﻔﮫ او ﺗﻐﯿﺮ اﻟﻌﻼج‬

، ‫اﻟﺤﺴﺎﺑﺎت ﻛﺎﻧﺖ ﻗﻠﯿﻠﺔ وواﺿﺤﺔ ﯾﻌﻨﻲ ﻣﺎ ﯾﺤﺘﺎج ﻟﮭﺎ وﻗﺖ‬

Red man syndrome >> vancomycin


Antidote for vasopressor
Antidote for opiates

Vancomycin through

Second line antiepliptic

herbal used for anti glycemic


a)ginseng.
b) aloe.
c) garlic
‫اﻧﺎ اﺧﺘﺮت ﺟﯿﻨﺴﻨﻖ‬

- 10 days old boy


a) infant. b) neonate. C) child

infant
organophosphate poisoning:
Atropin
Benztropine toxicity:
physostigmine
Patient with acidity and osteoporosis:
calcium carbonate
Antidote for Apixaban:

Idarucizumab
In USP, What used in parenteral preparation:
sterile water for injection
Cause steven johnson syndrome:
carbamazepine
Anergy in immunology :
describes a lack of reaction by the body's defense mechanisms to foreign substances
Omeprazol counseling :
Take before breakfast
Levothyroxine dose in pregnant :
Increase
Omeprazol and clopidogrel:
Cyp 2C19
codeine to morphine cause toxicity :
Ultra rapid
PIOGLITAZONE toxicity :
Liver
Vildagliptin toxicity :
quinine antimalarial and also use :
Counseling for Patient take orlistat :
Take multivitamin
Drug increase gout :
‫ ﻣﺮﺗﯿﻦ ﺟﺎء اﻟﺴﺆال ﺑﻄﺮﯾﻘﺔ ﻣﺨﺘﻠﻔﺔ وﻛﻠﮭﺎ اﺧﺘﺮت‬Diuretic
Warfarin interaction :
‫ أﺳﺄﻟﺔ وﻛﻠﮭﺎ ﺳﮭﻠﮫ ﻣﺮه ﯾﺴﺄل ﻋﻦ ال‬10 ‫ﺟﺎت أﻛﺜﺮ ﻣﻦ‬
INR ‫ وﻣﺮه ﻋﻦ ال‬pregnant ‫وزي ﻛﺬا‬
Pregnant women with HIV:
zidovudine
meningococcal vaccine:
‫ﻣﺘﻰ ﯾﻨﺎﺧﺬ اول ﻣﺮه ﻋﻨﺪﻧﺎ ﺑﺎﻟﺴﻌﻮدﯾﺔ‬
epimers and enantiomers: Structure
Antibiotic cause QT prolongation :
Macrolide
ESPL drug of choice :
Carpabenem
MRSA drug of choice :
Vacom
rhythm control drugs:
Sotalol
dose of naloxone in pediatric
0.01 mg/kg
‫ اﻧﻲ ﺳﺆال ﺣﻖ ﻣﻨﺪوب اﻷدوﯾﺔ اﻟﻠﻲ ﺳﻮا إﻋﻼن ل‬prescription drug ‫وﺟﺎت ﻋﻘﻮﺑﺔ ﻋﻠﻰ اﻟﺸﺮﻛﺔ ﺑﺴﺒﺒﮫ‬
‫ﻣﻦ ﺿﻤﻦ اﻻﺧﺘﯿﺎرات ان اﻟﻘﺎﻧﻮن ﺑﺎﻟﺴﻌﻮدﯾﺔ ﯾﻤﻨﻊ ذا اﻟﺸﻲ واﺗﻮﻗﻊ ھﻮ اﻟﺠﻮاب اﻟﺼﺢ‬
‫ ا ﯾﻘﻮﻟﻚ اﻟﺪﻛﺘﻮر ﺻﺮف ﻟﺒﯿﺸﻨﺖ‬Zolpedim
‫ ﻛﻢ ﯾﺒﻘﻰ ﻟﮫ‬refill
A. 1 refiil
B. 5 rifill
C. No rifiil
If IV no Rifill
‫ ﺳﺄل ﻛﻢ‬kilocalorie 1 ‫ﻓﻲ‬gm ‫ ﻣﻦ‬dextrose
For leg ‫ﻣﺪري وﺷﻮ‬
Melatonin

which is of them structure is 1,25 dihydroxycholecalciferol ?

Patient ‫ ﺟﺎء ﻟﻠﻤﺴﺘﺸﻔﻰ ﻛﺎن ﻣﺘﺴﻤﻢ ﻣﻦ دواء وﻋﻄﻮه‬benzodiazepines ‫ ھﺬا ﯾﻌﻨﻲ اﻧﮫ ﻣﺘﺴﻤﻢ ﻣﻦ ؟‬، ‫ﺑﺲ ﻣﺎﺗﺤﺴﻦ‬
Opioid
Coccain
Flumenzol
‫ ﺣﻄﯿﺖ اوﺑﻮﯾﺪ‬..
Girl want take isotretonin 1 tab for 3 month ?
‫وش اﻓﻀﻞ ﺷﻲء راح اﻗﻮﻟﮫ ﻟﮭﺎ ؟‬
‫ﺗﺎﻛﻞ ﻋﻠﻰ ﻣﻌﺪه ﻓﺎﺿﯿﮫ ؟‬
‫ ﻋﻠﺐ ؟‬٣ ‫ﺗﺎﻛﻞ ﻛﻞ اﻟـ‬
‫✅ او ﺗﻮﻗﻔﮫ ﺑﺸﮭﺮ ﻗﺒﻞ ﺗﺤﻤﻞ ؟‬

‫ﻟﻤﺎ ﺗﺠﯿﻨﻲ ﺑﻨﺖ ﺗﺒﻲ ﻋﻼج ﻟﺤﺒﻮب اﻟﺸﺒﺎب وش اﻋﻄﯿﮭﺎ ؟‬


‫ﻣﻦ ﺿﻤﻦ اﻟﺨﯿﺎرات‬
‫‪Hydrocortisone 1%‬‬
‫✅ ‪Hydro quinine‬‬
‫‪Facial sapon‬‬
‫‪..‬ﻣﺪري وش ﻛﺎن اﻟﺨﯿﺎر اﻟﺮاﺑﻊ‬

‫اﻟﻤﺪﯾﻜﺎل رﯾﺐ اﻋﻄﻰ دﻛﺘﻮر دواء ﺑﺒﻼش‬


‫ﻛﯿﻒ اﻟﻤﻔﺮوض ﯾﺴﻮي اﻟﺪﻛﺘﻮر ؟‬

‫ﯾﻌﻄﯿﮫ ﻋﻠﻰ طﻮل ﻻي ﻣﺮﯾﺾ‬


‫✅ ﯾﺤﺘﻔﻆ ﻓﯿﮫ ﺑﻐﺮﻓﺘﮫ‬
‫ﯾﺤﺘﻔﻆ ﻓﯿﮫ ﻓﯿﮫ ﻏﺮف اﻟﻨﯿﺮس‬
‫او ﯾﻌﻄﯿﮫ اﻟﺼﯿﺪﻟﯿﮫ اﻟﺪاﺧﻠﯿﮫ‬

‫‪ ng to mg‬ﺟﺎﻧﻲ ﺗﺤﻮﯾﻼت ﻛﺜﯿﺮ ﻣﻦ‬


‫ﻣﺎﯾﻜﺮو ﻗﺮام ‪Or ng to‬‬
‫ﺳﮭﻞ ﻣﺮة ﺑﺲ اﻋﺮﻓﻮا ﻛﯿﻒ ﺗﻄﺒﻘﻮﻧﮫ ‪ IBW‬ﺿﺮوري ﺗﻌﺮﻓﻮﻧﮭﺎ ‪ ،‬ﺟﺎﻧﻲ ﺣﺴﺎب‬
‫ﺑﺎﻗﻲ اﻟﺤﺴﺎﺑﺎت طﺮﻓﯿﻦ ﻓﻲ وﺳﻄﯿﻦ ‪ ،‬اﻻ ﻣﺴﺄﻟﺔ وﺣﺪة ﻣﺎﻋﺮﻓﺖ ﻛﯿﻒ اﺣﻠﮭﺎ ‪ ،‬ﻣﻮﺟﻮده ﺑﺎﻟﺰﺗﻮﻧﮫ ﺑﺲ ﻣﻐﯿﺮﯾﻦ اﻟﻤﻄﻠﻮب ﻋﺸﺎن ﻛﺬا ﺿﻌﺖ‬
‫‪ ..‬ﻓﯿﮭﺎ ‪ ،‬ﺑﺼﻮرھﺎ ﻟﻜﻢ ان ﺷﺎءﷲ‬

‫؟ ‪ ulcer colititis‬وش اﻓﻀﻞ ﻣﺴﻜﻦ ﯾﻌﻄﻰ ﻟﻤﺮﯾﺾ ﻛﺒﯿﺮ ﺳﻦ ﻋﻨﺪه‬


‫اﺳﺒﺮﯾﻦ‬
‫ﺑﺮوﻓﯿﻦ‬
‫ﺑﺎرﺳﯿﺘﻤﻮل ‪ +‬ﻛﻮدﯾﻦ‬

‫‪ orthostati hypertension‬وش ﻣﻦ ھﺎﻻدوﯾﮫ راح ﯾﺴﺒﺐ‬


‫‪Prazosin‬‬
‫‪Lisinopril‬‬
‫ﺑﺎﻗﻲ اﻟﺨﯿﺎرات ﻧﺴﯿﺖ‬

‫ﻣﺮﯾﻀﮫ ﺳﻜﺮ ﺟﺎت ﻋﻨﺪك ﺗﺸﺘﻜﻲ ان وزﻧﮭﺎ زاد ‪ ،‬وش اﻓﻀﻞ ﺷﻲء ﺗﻌﻄﯿﮭﺎ ﻣﺎﻛﺘﺒﻮا ان ﻋﻨﺪھﺎ ھﺎﯾﺒﺮﺗﻨﺸﻦ ﺑﺲ ﺑﺎﻟﻼب ﺗﯿﺴﯿﺖ ﻛﺎن ﻣﺮﺗﻔﻊ‬
‫‪٩٠/١٤٩‬‬
‫‪Insulin‬‬
‫‪Lisnopril + liraglutide‬‬
‫‪Liraglutid‬‬

‫‪ + rotavirus‬طﻔﻞ ﻋﻤﺮه ‪ ٤‬ﺳﻨﻮات ﺻﺎر ﻋﻨﺪه‬


‫وش ﺗﺴﻮي ؟‬
‫‪acyclovir‬‬
‫‪Rehydration oral‬‬

‫‪ fingolimod‬ﺟﺎﻧﻲ ﻣﺘﻰ اوﻗﻒ دواء‬


‫ﻟﻠﻲ ﺗﺒﻲ ﺗﺤﻤﻞ‬
‫✅ ﺷﮭﺮﯾﻦ ؟‬
‫ﺷﮭﻮر ؟ ‪٦‬‬

‫‪ ..‬ﺟﺎﻧﻲ ﯾﻤﻜﻦ ‪ ٣‬اﺳﺌﻠﮫ ﻋﻦ اﻟﻜﻮﺳﺖ ‪ ،‬اﻏﻠﺒﮭﺎ رﯾﻘﯿﻮﻟﯿﺸﻦ واﯾﺜﯿﻜﺲ اﻻﯾﺜﯿﻜﺲ اﺣﺲ اراء ﺷﺨﺼﯿﮭﺎ‬
‫ ﻣﺮﯾﺾ ﯾﺸﺘﻜﻲ ﻣﻦ‬dark stole
‫وش اﻟﺪواء اﻟﻠﻲ ﺳﺒﺐ اﻟﻤﺸﻜﻠﮫ ؟‬
‫ طﺒﻌًﺎ‬ferruse sulfate

‫ وش اﻟﺪواء اﻟﻠﻲ راح ﯾﻘﻠﻞ‬warfarin and NRI


Erethromycin
Rifampin
‫وادوﯾﮫ ﻏﺮﯾﺒﮫ ﺷﻮي ﺣﻄﯿﺖ رﯾﻔﺎﻣﺒﯿﻦ ﻣﺪري اذا ﺻﺢ‬..

‫ ﺳﺄﻟﻨﻲ ﻋﻦ اﻟﺒﻜﺘﯿﺮﯾﺎ اﻟﻠﻲ ﺗﺠﻲ ﻋﺎﻟﻈﻔﺮ وﻧﺎﺳﯿﮫ ﺻﺮاﺣﺔ‬..

Prophylaxis enoxabarin dose 40 once daily ..


‫ ﻛﻢ ﯾﺎﺧﺬ اﻧﻮﻛﺴﺎﺑﺎرﯾﻦ ؟ طﺒﻌًﺎ‬٨٠ ‫وﺑﺮﺿﻮ ﺳﺆال ﺛﺎﻧﻲ ﻣﺮﯾﺾ وزﻧﮫ‬
80 BID

‫اﯾﮫ وﺟﺎﻧﻲ ﻣﻦ اﻟﮭﯿﺮﺑﺎل وش اﻟﻠﻲ ﻣﻤﻜﻦ ﯾﻮزد اﻟﻔﯿﺰﯾﻜﺎل ؟‬


‫ اﻟﺨﯿﺎرات‬ginger, ginseng , chamomile ‫ وﺷﻲ ﯾﺒﺪأ‬Ench ‫ﺣﻄﯿﺘﮫ ھﻮ ﻻن ﻋﻠﻰ ﻋﻠﻤﻲ وﻻ ﺧﯿﺎر ﻣﻤﻜﻦ ﯾﻨﻔﻊ ﻣﻘﺎرﻧﮫ ﺑﺎﻟﺒﺎﻗﻲ‬..

‫ ﺟﺎﺗﻨﻲ اﺧﺘﺒﺎرات ﻋﻘﺎﻗﯿﺮ اﻟﻠﻲ‬fehling test ‫اذا ﺻﺎر ﺑﻮزﺗﯿﻒ وش ﯾﻌﻨﻲ ؟‬

‫ رﺳﻤﺔ‬ECG ‫اﻓﮭﻤﻮھﺎ زﯾﯿﯿﻦ‬

‫اﻋﺮﻓﻮا اﺳﻤﺎء اﻻﻣﺮاض ﺣﻘﺖ اﻟﺜﺎﯾﺮوﯾﺪ ﻣﻮھﺎﯾﺒﻮ وﻻ ھﺎﯾﺒﺮ ﻻ اﻟﺜﺎﻧﯿﮫ‬


1- Which of the following statements best describes oxytocin?
A. High doses may lead to hypertension
B. Steady state is reached within 5 to 10 min
C. Uterin hyper stimulation is associated adverse effects
D. Higher doses can lead to vasopressin receptor related urinary retention ✅

2- which of the following class Iv anti arrhythmia?


A. Flecainide
B. Verapamil ✅

3- which of the following stimulation a1 adrenoceptors?


A. Increased heart rate
B. Elevated blood pressure ✅

4- Itraconazole capsules require acidic ph dissolution and absorption, which of the following approriate
advice to the oral absorption the drug?
A. Take it with food ✅
B. Take it on empty stomach

5- which of the following used describe the pharmaceutical technique grinding insoluble substance fine
powder while wet ?
A. Trituration
B. Levigation ✅

6- which of the following can use with HCG+ ?


A. Levothyroxine ✅
7- boy brought to Emergency with complaints of inability to open eyes and difficulty in breathing with
history snake bite ?
A. Give anti snake venom plus Neostigmine ✅

8- routes of drug administration provide 100% bioavailability?


A. Intravenous injection ✅

9- Meniere disease ? betahistine ✅

10- dose dependent? Phenytoin ✅

11- which of the following studies drug shelf life ?


A. Stability studies ✅

12- SABA inhaler cause SE ?


A. oral candidiasis
B. Tachycardia ✅
C. Hyperglycemia

13- structures ciprofloxacin ? Antibacterial activity especially G - negative ✅

14- the effect alkaline buffers in aspirin?


A. Reduce rate absorption of aspirin and less readily absorbed ionic form ✅

15- amiodarone cause ? Hyperthyroidism ✅

16- A question about Epoetin Use it with kidney patient


I do not remember😪

17- ratio mass to the volume? Absolute density ✅

18- old patient use ? Triazolam✅

19- patient with history of pneumonia ‫وﻋﻨﺪه ﻣﺸﺎﻛﻞ ﻛﺜﯿﺮ ﻣﻦ ﻋﺪوات ﻓﯿﺮوﺳﯿﮫ‬give vaccines?
A. Pneumonia + meningitis
B. Flu
C. Pneumonia + flu vaccine ✅

20- side effect anticoagulant? Manjor bleeding✅

21- active metabolite of primidone ? Phenobarbital ✅

22- ‫ﻓﯿﮫ ﺳﺆال اﯾﺶ اﻓﻀﻞ ﺗﺼﻨﯿﻊ ﻻدوﯾﺔ اﻟﺘﺎﺑﻠﺖ ﺗﺨﻔﻲ اﻟﻄﻌﻢ اﻟﺴﻲء‬
A. Inter coating ✅ not sure
B. Capsule
C. Sublinqual
23- bisphosphonate ? Osteonecrosis the jaw✅

24- finasteride ? Teratogenic ✅

25- method for sterilizing ophthalmic solution ?


A. Autoclaving for 15 min
B. Autoclave for 30 min
C. Membrane filtration 0.2 ✅

26- plate method? DM ✅

27- ‫ اﯾﺶ ﻣﻌﻨﺎه ؟‬2.5 ‫ ﺳﺎﻋﺎت ﺳﺤﺒﻨﺎ ﻣﻨﮫ وﻟﻘﯿﻨﺎ ﻧﻔﺲ اﻟﺘﺮﻛﯿﺰ ﻣﺎﺗﻐﯿﺮ‬٥ ‫ ﻣﻦ اﻟﺪﯾﺠﻮﻛﺴﻦ وﺑﻌﺪ‬2.5 ‫ﺳﺆال ان طﻔﻞ اﺧﺬ‬
‫ ﻧﺴﯿﺖ اﯾﺶ اﺧﺘﺮت ﺑﺲ اﻟﺨﯿﺎرات ﻋﻦ‬VD and half life

28- ‫اﻟﻔﺼﻞ ﺑﯿﻦ ﻗﻄﺮﺗﯿﻦ‬


A. 5 sec
B. 5 min ✅

29- ‫اﯾﺶ ﻣﻦ ادوﯾﺔ اﻟﺴﺎﯾﻜﻮ ﻧﺴﺘﺨﺪﻣﮫ ﻓﻲ اﻻﻋﺮاض اﻟﺒﻮﺳﺘﭫ‬


Haloperidol ✅

30- ‫ﻛﯿﯿﺴﺲ طﻮووﯾﯿﯿﯿﻞ ﺑﻌﺪ اﻟﻔﺤﻮﺻﺎت اﻛﺘﺸﻔﻨﺎ ﻋﻦ اﻟﻤﺮﯾﺾ ﻣﻼرﯾﺎ اﯾﺶ ﻣﻦ اﻟﺘﺎﻟﻲ ﻋﻼج ﻟﻠﻤﻼرﯾﺎ‬
A. Quinine ✅

31- ‫ اﺳﺌﻠﮫ ﻋﻦ ﺗﺎﺛﯿﺮ‬٥ ‫ﺟﺎﺗﻨﻲ ﻣﻤﻜﻦ‬


inhibitor and inducer ‫ﻋﻠﻰ اﻻدوﯾﮫ‬

32- druge for acute gute ? Indomethacin✅

33- pinworm infection? Albendazole✅

34- best represent the intravenous?


IV drugs transmited the high risk infection✅

35- Reliability definition ?

36- Which of the following describes Mode in statistics? ‫ﺗﺨﺘﺎرون اﻟﺮﻗﻢ اﻻﻛﺜﺮ ﺗﻜﺮار‬

37- ‫ ﺟﺎﻧﻲ ﺳﺆاﻟﯿﻦ ﻋﻦ‬therapeutic index


LD and ED ‫ﻣﻦ اﻟﺮﺳﻤﮫ ﺗﺤﺴﺒﻮﻧﮭﺎ ﻧﻔﺲ اﻟﻲ ﺑﻤﻠﻒ اﻟﻤﺴﺎﺋﻞ‬

38- which of the following anesthetic drug? Isoflurane ✅

39- doxycycline 2 hours before or 4 hours after? iron ✅

40- which of the following timeline where symptoms start to occure type 1 DM ?
A. When 70% of the isolets of the pancreases ✅

41- furosemide? Ototoxicit ✅


42- in marketing price & place & product? Promotion ✅

43- in cytotoxic Type 2 ? IgG

44- hotel And travel for treatment? D Non medical ✅

45- bacteria in Osteomyelitis? E. coli ✅


‫ ﻣﺎﻛﺎن ﻣﻮﺟﻮد‬staphylococcus aureus

46- duration for treatment osteomyelitis ?


A. 10 days
B. 7 days
C. 4 to 6 weeks ✅

47- zopideme ? No refil ✅

48- electron transport chain? Inner mitochondrial ✅

49- atropine use for ?? And class for artopine ?

50- price of product and demand dose? Elastic ✅

51- ‫ﺟﺎت اﻟﺮﺳﻤﮫ ﺗﺒﻊ اﻻﺳﮭﻢ ﻛﻼﺳﺎت ادوﯾﺔ اﻟﻀﻐﻂ‬

52- ‫ اﻋﺮﻓﻮ‬RNA to proteins translation


And DNA trancription ‫ﺟﺎء ﺳﺆاﻟﯿﻦ ﻋﻠﯿﮭﺎ‬

53- Kilocalories in 1 gm dextrose?

54- Hepatitis A in travel

55- in class B COPD which best treatment ?


A. Albutrol
B. Ipratropium
C. Tiotropium

56- structures for Enantiomers

57- ‫ اﻋﺮﻓﻮ اﻋﺮاض‬Zolpidem ‫ﻋﻠﻰ اﻟﻤﺮﯾﺾ اﻟﻲ ﯾﺴﺘﺨﺪﻣﮫ‬

58- ‫ﻋﻼج اﻟﻤﻼرﯾﺎ ﻓﻲ اﻟﺴﻌﻮدﯾﮫ ؟‬


artesunate ✅

‫ ﻓﻲ ﺳﺆال ﺟﺎﻧﻲ ﻛﯿﺰ وﺟﺎب اﻋﺮاض اﻧﮫ واﺣﺪ ﯾﺸﻮف اﻟﻮان اﺧﻀﺮ واﺣﻤﺮ اﻋﺮاض ﻋﻼج‬ethambutol ‫ ﺣﻖ‬TP ‫وﯾﺴﺄل وش ﻣﺴﺒﺒﮭﺎ‬
‫ﻣﻦ اﻟﺨﯿﺎرات‬

1. Vancomycin trough sever? 15-20


2. Antidone for opiod (case)
3. Css law = F*MD / Cl * T
‫ و ﻣﻄﻠﻮب ﺗﻄﻠﻊ ال‬T1/2 ‫ ﻣﻦ ال‬Cl ‫ ﺳﺎﻋﺎت‬10 ‫ﻋﺸﺎن ﺗﺴﺘﺨﺪﻣﮭﺎ و ﺗﻌﺮف ﺑﺎﻗﻲ ﻗﺪ اﯾﮫ ﻣﻦ اﻟﺪوا ﺑﻌﺪ‬
4. Cohort prospective quest
5. ‫ﻣﻌﻈﻢ اﻟﻤﺴﺎﺋﻞ وﺳﻄﯿﻦ ﻓﻲ طﺮﻓﯿﻦ‬
5. Corrected phenytoin lvl
6. Opioid constipation >> ADR
7. Drug cause red man syndrome >> vanco
8. Pregnant >> switch to methyl dopa
9. Agar plate (sensitivity)
10. Therapeutic index graph >>
TD50 / ED50
11. TD50? From graph
12. Add to metformin for diabetes with heart failure? Gliflozin
13. Met + sitaglip +... Monitoring? Renal function
14. Nano micro milli conversions
15. % W/W concetration
16. No of tablets
17. No of bottles for syrup
18. Olive oil + vit A + water i choose emulsion
19. Teaspoonful ‫ﻣﮭﻤﮫ‬
20. Dietary for hypertriglycerid? Fish oil
21. Gemfibrozil + statin >> rhabdo
22. Medication regulation >> sFDA
23. Certificate to foreiners >> SCFHS

24. Strucure ibuprofin

25. Aspirin structure

26. Decrease flushing >> aspirin

27. Drug induce c.difficle >> clindamycin


28. C.difficle >> wash with soap and water
29. P. Aurignosa pt take ceftazedime in hosp and need oral ( p.aur susceptible to cipro levo ceftaz... )

Only cipro was in choices

33. Patient counceling for nitroglycerin>> keep in original containers and keep them with u all the time
34. Doxycyclin and adapalene heart burn? >> take with penalty of water

*17 ‫*اﻛﺘﻮﺑﺮ‬
‫اﻟﺤﺴﺎﺑﺎت‬
‫ﻓﯿﻨﻮﺗﯿﻦ ﻛﻮرﯾﻜﺖ دوز‬
VD
LD
Crcl
W/w %
1 Test for phamackintic renal funcatiob
- crcl
- Cr
2 SSRI to be SNRI should be ubder to :
3 lamotigine SE induce Rash
4 phenotyin SE gingival hyperplasia
5 ‫ ﺳﺄل ﻋﻨﺪ دواء ﻓﯿﮫ‬mac ‫ﻣﺎ اﺗﺬﻛﺮ اﺳﻢ اﻟﺪواء‬
6 alchool used in USP ?
10 ‫اﻟﻤﺮﯾﺾ اﻟﻠﻲ ﻣﺎﯾﺒﻲ ﯾﺎﺧﺬ اﻧﺴﻮﻟﯿﻦ‬
11 non - adherance type
12 vaccine (‫ اﻻﺳﺌﻠﺔ اﻟﻤﻜﺮره اﻟﺤﺞ واﻟﻮﯾﻨﺪو واﻻم ﺑﻮزﺗﻒ‬Hip B)
13 patient take warfarin INR 10 what we do ? Vitm K
14- amphemine MOA
15 - cimetdine with clopidgrel ( what do )
16 - second line in UTI fosfomycin, levo, cipro
17- candais tretemnt
18- 2 Q about analgisc in osterarthitrs
19 - insulin unit and typer add to patine on NP
20 - sensetive pic ( what is max sensetive)

‫ﺗﺬﻛﺮت ﺟﺎ ﺳﺆال اﯾﺶ ال‬antiemetic ‫ اﻟﻠﻲ ﯾﻨﻌﻄﻰ ﻟﻤﺮﺿﻰ ال‬parkinson ‫ اﻟﺠﻮاب‬dompiredone

Pt with chadsvasc more than 5 what is the management


-Zolpidem max dose in women: 5-10-15 mg
-Pt 40 year old no comorbidities want a trearment for insomnia options include: zolpidem,
diazepam,midazolam
-Anesthesia cause hepatic liver elevation: isoflurane or nitros oxide
-Dose and frequency of ceftriaxone in meningitis
- lots of questions about ecg
- Pt treated for uti with bactrim she is adhered no allergy or side effects to the medication but came
suffering from dysyuria and suprapubic pain, what is the action taken : use another agent
- Question with structures without names and asks what is the best option for women with menstural
pain: diclofenac
- Vitamin supplement taken with caution in pregnant women
- Therapeutic index calculation from graph
- Corrected phenytoin calculation
- Calculate cl in one compartment drug ‫ ﻣﺤﻄﻲ دوز و ﺟﺪول ﺗﺤﺴﺐ‬k‫ ﻣﻦ اﻟﺠﺪول‬k=-slope
- Infusions rate calculation
- Alot of Economic questions
- Type 1 sensitivity involves which Ig
- Test before taking finglimod
- Omeprazole councling
- Drug used in rate control
- Pt with hx of ischemia and Afib came with PE what is tge management, spironolactone or
hydrochlorothiazide or furosemide
- Pediatric pt came to ER suffrenig from‫ ﻧﺴﯿﺖ اﯾﺶ‬after ingesting castor oil bean what is the active
constituent of castor bean: i chose ricin
- Pt complain lack of effect from her asprin she keeps the medication in the bathroom closet what is the
cause of ditorioration of the medication: oxidation or hydrolysis?
-pt was prescriped drug x from her doctor for her osteoporosis the drug was prescribed for every week
and to take it and avoid eatin or laying down? Alendronate
-Pt with inr <2 despite warfarin dose of 10 mg po, what is the cause?
Gene vokpr1c and ‫ ﻧﺴﯿﺖ اﯾﺶ‬polymorphism , *decreased* intake of green vegetables, dose need to be
increased.

Pt with chadsvasc more than 5 what is the management

Pt prescribed amitriptylin and enteric coated asprin, what is the advantage?


-decrease absorption of amitriptylin
-increase absorption of amitriptylin
-delay absorption of amitriptylin
‫ﻧﺴﯿﺖ اﯾﺶ ﻛﻤﺎن‬

Betacarotene is precursor of which vitamin

‫ ﻓﻲ ﺳﺆال ﯾﻘﻮل ﻛﯿﺰ وﺟﺎب اﻋﺮاض اﻧﮫ واﺣﺪ ﯾﺸﻮف اﻟﻮان اﺧﻀﺮ واﺣﻤﺮ اﻋﺮاض ﻋﻼج‬ethambutol ‫ ﺣﻖ‬TP ‫وﯾﺴﺄل وش ﻣﺴﺒﺒﮭﺎ‬
‫ﻣﻦ اﻟﺨﯿﺎرات‬

‫ ﻛﺎن ﻓﯿﮫ ﺳﺘﺮﻛﺘﺸﺮز‬.. ‫ ﻣﺎ ﻛﺎن ﺳﮭﻞ وﻻ ﻛﺎن ﺻﻌﺐ ﻧﺺ وﻧﺺ وﺟﺎ ﻣﻦ اﻟﺘﺠﻤﯿﻌﺎت ﻛﺜﯿﺮ‬، ‫ اﻛﺘﻮﺑﺮ‬١٨ ‫ اﻟﯿﻮم ﻛﺎن اﺧﺘﺒﺎري‬، ‫ﻣﺴﺎء اﻟﺨﯿﺮ‬
‫ ﺳﺆال ﻛﺬا‬٣٠ ‫ او‬٢٠ ‫اﻋﺬروﻧﻲ اذا ﻓﯿﮫ 🤍 و ﷲ ﯾﻮﻓﻘﻨﺎ ﺟﻤﯿﻊ ﯾﺎرب 🤍 ﻛﺘﺒﺖ ﻟﻜﻢ اﻻﺳﺌﻠﺔ اﻟﻠﻲ ﺗﺬﻛﺮﺗﮭﺎ ان ﺷﺎء ﷲ ﺗﻔﯿﺪﻛﻢ 😬 ﯾﻤﻜﻦ‬
‫👍 اﺧﻄﺎء او اﻻﺳﺌﻠﺔ ﻣﺎ ﻛﺎﻧﺖ واﺿﺤﮫ و اي اﺳﺘﻔﺴﺎر اﻧﺎ ﻣﻮﺟﻮده‬

- 🛑 Longest insulin ?: glargine


- 🛑Antihistamine don’t cause sedation ?: fexofenadine
- 🛑Atenolol classification : b1 selective
- 🛑Sitaglptine ?: DPP-4
- 🛑Deep wound with unknown history vaccine ?: TD+ TIG
- 🛑Obese want orlistat ? Give with multivitamins
- 🛑Pregnant women with sever vomiting?‫ ﻛﺎن ﻓﯿﮫ‬domepridon ‫ و‬pyridoxine
- 🛑Phenytoin induced ? Gingival hyperplasia
- 🛑Antidote for benzodiazepine ?flumazenil
- 🛑Antidote for morphine? Naloxone
- 🛑Tuberculosis vaccine time ? At birth
- 🛑Ou TID? Both eyes 3 times daily
- 🛑Pt need 15 ml of syrup ? 1 tablespoon
- 🛑As needed abbreviation ? PNR
- 🛑Structure of : quinidine - aspirins
- 🛑Study design ? ‫ وﻓﯿﮫ اﺷﯿﺎء ﻛﺜﯿﺮه ﺟﺎﺑﻮھﺎ ﻣﻦ‬study design ‫ اﺳﺌﻠﺔ ﻛﻠﮭﺎ واﺿﺤﺔ‬١٠ ‫ﯾﻤﻜﻦ‬
- 🛑Economic: scenario with (monitory)? Benefit analysis
- 🛑Scenario with ( clinical) ? Effective analysis
- 🛑Scenario decreased mortality of cancer ? I chose CUA
- 🛑compared two drugs with same equivalent? CMA
- 🛑Colanzepin test ? ANC
- 🛑Regulation of practicing? MOH
- 🛑Scores of CHD2DS2-VASc ? ‫ وﻋﻨﺪه‬٧٦ ‫ ﺟﺎب ﺷﺨﺺ ﻋﻤﺮه‬CHF /HTN 4 ‫ ﻛﻢ ﻋﺪد اﻟﺴﻜﻮرز؟‬scores
- 🛑Bleeding with aspirin for colon cancer ? 3 months / 6 months / 5-10 years / more than 20 years I
chose (5-10) I’m not sure
- 🛑797 ? sterile
- 🛑Laminar flow hood contamination ? Equipment/people / I don’t remember , but I chose people😬
- 🛑Case: pt have sever pain due to sickle anemia ? Morphine
- 🛑Vitamin in high level maybe potential risk for pregnancy? Vitamin A
- 🛑Anesthetic for cardio toxicity ? Bupivacaine / lidocaine , I chose bipivacaine
- 🛑Case : pt take (mg) (linsopril) and other drugs I can’t remember,, what cause diarrhea? Mg
- 🛑Case : ‫ ﺷﺨﺺ ﻋﻨﺪه‬TB ‫ﺑﺲ ان اﻻﺧﺘﯿﺎرات ﯾﺎ ان ﺗﺨﻠﯿﮫ ﺑﺎﻟﻌﺰل او ﺗﺨﻠﯿﮫ ﺑﻐﺮﻓﺔ ﻋﺎدﯾﺔ ﻣﻊ 😂 وﻣﺪري وش ﺻﺎر ﻟﮫ ﺻﺮاﺣﺔ‬
‫اﻟﻨﺎس ؟ اﺧﺘﺮت اﻟﻌﺰل‬
- 🛑Plant use to treat vomiting ? Gengir
- 🛑Anticoagulant use for mechanical valve ? Warfarin
- 🛑Phase 4 in action potentials?
- 🛑QRS? Ventricular depolarization
- ‫ اﻣﺮأه ﻋﻠﻰ‬levothiroxine ‫ ﻷي‬،، ‫ دﻛﺘﻮر ﺗﺮوح ﻋﺸﺎن ﯾﻐﯿﺮ ﺟﺮﻋﺘﮭﺎ ؟ 🛑 ﺻﺎرت ﺣﺎﻣﻞ‬Obstetrician and Gynecologist/
Endocrinologist
- 🛑Allopurion dose for adjustment? 100mg /150mg/ 200 mg
- 🛑Vinca plant ? Vincristin
- 🛑DNA to mRNA ? Transcription
- 🛑Alcohol use in USP ?
- 🛑Sensitive antibiotic ? ‫ﺻﻮره‬
- 🛑Case : patient with high troponin and ST elevated? STEMI
- 🛑Case : 12 years old have penicillin allergy ?
- 🛑Bentonite ? Emulsifying agent /suspending agent

🛑🛑 Calculations 🛑🛑
🛑 mEq
🛑BMI
🛑Crcl
🛑LD
‫ وﻣﺎ ﺟﺎﻧﻲ ﻛﺜﯿﺮ ﺣﺴﺎﺑﺎت‬، ‫اﻟﺒﺎﻗﻲ ﻛﺎن ﺳﮭﻞ وﺳﻄﯿﻦ ﻓﻲ طﺮﻓﯿﻦ‬
- hep B +ve mother

-deep wound /minro

-reponsible for signal ➡SA node

-tb vacc

-adaptogenic plant

-rota virus therapy

-responsible color in fruit ➡falvonoid

-hypotonic /hypertonic

-hashimotas symptoms

-petri dish

-ECG

-lithium monitor

-vancomycin trough

-efficacy /potancy graph

Anti dote
Iron . Chlonrigic . Lithium
INR ‫ﺣﻖ اﻟﻮراﻓﺮﯾﻦ‬
‫ ﺟﺎﻧﻲ اﺳﺘﺮﻛﺸﯿﺮ ﺣﻖ‬CCB
Xalatan ‫ﺣﻖ ﺟﻠﻜﻮﻣﺎ‬
‫ ﺟﺎﻧﻲ ﺳﻮال اﯾﺶ ھﻮ اﻟﻠﻲ ﯾﻨﻔﻊ ﯾﺴﺘﺨﺪم‬anti inflammation ‫ و‬analgesic ‫ ﻛﺎن ﺿﻤﻦ ﺧﯿﺎرات‬ibuprofen
‫ ﺟﺎﻧﻲ ﻓﻮق ارﺑﻊ اﺳﺎﻟﮫ ع‬arrthmia
‫ ﺟﺎﻧﻲ ﺳﻮال‬double bound ‫ ﺟﺎوﺑﺖ‬C4h4
Smoker drug interaction with of olanazpine
‫ ﺟﺎﻧﻲ ﺳﻮال ال‬contraceptives ‫ ﻣﻊ ال‬ethromcine
‫ ﺟﺎﻧﻲ ﺳﻮال اﯾﺶ اﻛﺘﺮ ﻧﻌﻤﻞ اﻟﮫ‬monitoring of coagulation
IV UfH
‫ ﻓﯿﮫ ﻣﺮﯾﺾ ﻋﻨﺪه‬consptiaion ‫ ﻣﻦ ال‬morphine ‫ ﺷﻮ ﻧﻌﻄﯿﮫ ﻛﺎن ﺿﻤﻦ ﺧﯿﺎرات‬docusate and senna
Hospital formulary ‫ ﻣﯿﻦ ﻣﺴﺆول ﻋﻠﯿﮫ‬p&t
Anti hyper tensie. Hazard for child
‫ ﻓﯿﮫ ﻣﺮﯾﺾ ﻋﻨﺪه‬consptiaion ‫ ﻣﻦ ال‬morphine ‫ ﺷﻮ ﻧﻌﻄﯿﮫ ﻛﺎن ﺿﻤﻦ ﺧﯿﺎرات‬docusate and senna

‫ ﺟﺎﻧﻲ‬Max dose of prednisone

‫ اﺟﺎﻧﻲ ﺳﻮال ع‬colazpine monitor


‫ ﺟﺎﻧﻲ ﺳﻮال‬prevent of vomiting p pregnant

‫ اﻻﺳﺘﺮﻛﺸﺮ ھﻮ ﺟﺎﺑﻲ اﯾﺶ ھﻮ‬dihydropyridine

You might also like